Está en la página 1de 66

Evidence (8/18/08 & 8/20/08) - General Principles of Relevance Movie: about brothers where Dell murdered his brother

r William (Bill) b/c he was sick i.e. mercy killing (prosecution) OR that he died in his sleep (defense) o Delbert waived his rights and did not know what that meant; he gave a confession to the police o Community believes that the man is innocent o ALWAYS identify the evidence being tested Relevance does not usually arise with direct evidence, usually arises with circumstantial. Direct evidence: evidence that actually asserts the fact that you are trying to establish. (i.e. eyewitness testimony) Circumstantial evidence: evidences from which the fact-finder can infer from that which is true. (i.e. I heard the gun and saw him run awaycan be inferred, but not known) Probability: It must tend to make a fact more or less probable. Evidence must be probative of a material fact. That is, the evidence must have a tendency to make the existence of that factmore probable or less probable than it would without the evidence. Material: The fact must be material, i.e. it must bear on a fact that is of consequence to the determination of the action (lost wages are not a factor in a criminal trial but are definitely material in a wrongful death suit) To decide whether an issue is material, do not look to the evidence rules for an answer, but to the substantive law. (Substantive Law: think of the elements of the crime) MUST have BOTH materiality and probability, or it is NOT relevant. FRE 401: Definition of Relevant Evidence Relevant evidence means evidence having any tendency to make the existence of any fact that is of consequence to the determination of the action more or less probable that it would be without the evidence. Evidence must be probative of a material fact. The evidence must have a tendency to make the existence of that fact more probable or less probable than it would be without the evidence. i.e. testimony from an employer about the deceaseds salary is surely probative but not necessarily material. If an item of evidence tends to prove or disprove any proposition, it is relevant to that proposition. FRE 402: Relevant Evidence: Admissibility Evidence that is relevant IS admissible Evidence that is not relevant is NOT admissible. Exception: U.S. constitution, U.S. statute; FRE; other rules FRE 403: Exclusion of Relevant Evidence on Grounds of Prejudice, Conclusion or

Waste of Time Although relevant, evidence may be excluded if its probative value is substantially outweighed by the unfair prejudice, confusion or misleading of the jury or by considerations of undue delay, waste of time, or needless presentation of cumulative evidence. (Exceptions to the FRE that relevant evidence is always admissible) Under substantially outweighed has to be more, CAN NOT be at the same level or slightly over. Unfair prejudice is that it has to be inherently prejudicial Does not only apply to but to as well 403 should always come at the end of your analysis. There is always a 403 argument. (i.e. would say the probative value outweighs the prejudicial effect. OR would say the prejudicial effect outweighs the probative value.) The evidence may be relevant but can still be excluded based on 403. This FRE is based on the judges discretion so CAN ONLY be reviewable for the abuse of discretion FRE 101: Scope These FREs govern proceedings in the courts of the United States and before the United States bankruptcy judges and the United States magistrate judges, with exceptions stated in 1101. Purpose and Construction These FREs are construed to secure fairness in administration. Elimination of unjustifiable expense and delay, and promotion of growth and development of the law of evidence to the end that the truth may be ascertained and proceedings justly determined Preliminary Questions: Relevancy conditioned on fact (a) Preliminary questions concerning the qualification of a person to be a witness, the existence of a privilege, hearsay, or the admissibility of evidence shall be determined by the court (judge not the jury) FREs do not apply to bond hearings or sentencing hearings. (b) Conditional Relevance: When the relevancy of evidence depends upon the fulfillment of a condition of fact, the court shall admit it upon or subject to, the introduction of evidence sufficient to support a finding of the fulfillment of the condition. i.e when the relevancy of one piece of evidence depends on another piece of evidence **court makes determination** (c) Hearings on admissibility of confessions shall in all cases be conducted out of the hearing of the jury. Hearings on other matters shall be so conducted when the inters of justice are required (d) The accused does not by testifying in another preliminary matter become subject

FRE 102:

FRE 104:

to cross-examination as to other issues in the case (e) FRE does not limit the right of any party to introduce the jury evidence relevant to weight or credibility 1) Probativeness Problem 1.1 Wife asks where is the body? Defense should object as to relevanceSince the prosecutor would likely argue that the chain of inferences shows guilt...a normal person would have asked what murder? Who was murdered? Not where is the body. Her statements tend to make it more probable that she had knowledge of the crime, thereby making it more probable that her husband committed the murder. Statements were admitted as evidence. If not objected to now, it cannot be appealed later. Problem 1.2 E testifies against . brings forth a witness, M, who says that while in prison, E told him that he was going to falsely incriminate . Attack on Es credibility. on cross-examination asks if these guys are members of a secret prison society. should object to these questions, saying that whether M and were members of a prison gang is not relevant. can argue credibility. could also argue that this evidence is prejudicial to the jury under FRE 403. The state would argue that this questioning is relevant for it could show that M would lie for if they were in a secret society where members lie and kill for each other. It is important for jury to know that was in a gang that lies and cheats for each other, b/c the jury determines the credibility of a witness. It was allowed by the trial court. Problem 1.3 Polygraph examiner is not allowed to testify as to the results of s polygraph, but explained how the polygraph test works and that said go ahead Doc hook me up wants to allow this evidence in, since it shows that willingly took a polygraph test without hesitation. Most people know that the result of a polygraph are inadmissible. Prosecutor should object on the grounds that these were his statements before the test and are not indicative of the results. In addition, defense attorney arranged for the test and could have spoken to him before hand. Evidence was excluded. 2) Materiality Problem 1.4 s failure to KNOW that her crime of assault and battery was punishable by more than a year in prison and thereby prevented her from carrying a firearm. The statute has no knowledge requirement. Defense would like to keep this in b/c she did not know until the arrest. Prosecutor would likely argue that her knowledge is not materialwhether she knew or not, she was breaking the law (strict liability). knowledge is irrelevant Problem 1.5 3 went drinkingafter reports of a drunk driver, the police found the car on the side of the road with the two front passengers dead from gunshot wounds through the forehead. The third passenger was found alive in the backseat. His gun was found on the floor, and gunshot residue was found on his hands. The state charged him with deliberate homicide, of knowingly and purposefully killing. Defense seeks to admit evidence of his level of intoxication. Defense argues that (it is probable) he could not have acted knowingly or purposefully since he was highly (.36) intoxicated. The prosecution will likely argue that voluntary intoxication is not a defense, i.e. his level of intoxication is not material, therefore not relevant.

Problem 1.6 Prosecutor tries to admit evidence of absence of a gun in a self-defense case involving a police officer. The defense tries to keep the evidence out. Argument for defense: the police officer believed the victim to be carrying a gun and acted relevantly. The prosecution would argue that there was no gun present, and therefore the officer was not defending himself from an actual threat. Prosecution probably would not be able to bring in evidence that there was no gun, therefore no threatnot relevant! Since it is a subjective, whether that person felt he was in dangerif he felt he was, then self defense was reasonable with or without a gun. Problem 1.7 was charged with murdering his wife. The hoped to admit evidence that the victim told Brown that she going to tell their oldest son that his real father was Brown and not the defendant. The wants this evidence to come in, b/c it shows the s motive to commit the crime. Issue: Whether the evidence concerning paternity was relevantonly relevant if the knew about it (conditional relevancy). From the facts, DNA, references to her son, the judge allowed the evidence in. ROL Huddleston Standard: the court has to find by sufficient evidence that a jury could find by a preponderance of the evidence that the knew of the paternity, before the evidence can come in (In order to get rid of this issue is by filing a motion in limine before the trial even begins) Jury must find beyond a preponderance of the evidence. 3) Photos and Other Inflammatory Evidence Problem 1.8 Was the photograph of a semi-automatic weapon that fired more than once for the state, but not for the defendants expert relevant, when the photo did not show whether the weapon was dirty, malfunctioning, etc. The photo did not show the probative value, and was unfairly prejudiced since the photo showed various other weapons not belonging to the defendant. The photo showed nothing of the guns interior. However, if you could tell that the gun was dirty. The expert should have been able to crop the photograph, so that the other weapons were not displayed. Similar to Itos decisionnot allow all, but will allow some. The Fuhrman Tapes: Fuhrman testified at trial that he had not used the word nigger in the past 10 years. The defense then sought to admit tapes where Fuhrman used the word nigger 41 times. The state argued that the use of such an inflammatory word to a jury containing 9 African Americans would severely discredit their star witness. Judge Ito allowed four of the usages in, but held that the probative value of the others was severely outweighed by potential prejudice. 4) Evidence of Flight: Problem 1.9 Despite s objections, state was allowed to offer evidence that the had fled from the scene of the crime. When questioned by his attorney, stated that he fled, b/c he was scared he would be arrested, mostly due to his priors. Prejudice to , as most jurors would think only guilty parties run from the police. It is prejudice just to bring up s criminal record. On appeal, the court should find no abuse of discretion by the judge in allowing evidence of flight in. The time lapse between the crimes makes it less prejudicial so the evidence would be let in. Problem 1.10 In this case the ran based on another reason, so he has a stronger case. The flight should have not been introduced b/c it has nothing to do with the attempted murder of the victim. As , the argument would be that he had a guilty mind and knew that the police were coming after

him for the attempted murder. The police were not close to know what was in the jar so based on his flight it was based on his previous criminal act. Here in this case its highly prejudicial to the s case b/c he stayed within the city. This actually can confuse the issues for the jury. Problem 1.11 If the state can get in evidence of flight (as being probative of guilt), why cant the defense admit evidence of people staying put. Solid majority states that evidence of staying put is not relevant or admissible in trial. Probability Evidence: People v. Collins Very rarely can one admit probability evidence to prove likelihood. The prosecutor had no case and tried to impress jurors through the use of made up mathematical statistics! Here, the prejudice outweighed the probative value. Probability evidence can be admitted (i.e. DNA evidence), but it must be reliable (8/25/08 & 8/27/08) - The Specialized Relevance FREs These FREs are used for certain limited purposes (based on 403 balancing test) Route of Admissibility (Taking evidence of subsequent remedy as example) FRE 401 if evidence has any tendency to make a material fact more or less probable FRE 402 then it is admissible unless otherwise barred if evidence is of subsequent remedy FRE 407

and is offered to prove negligence, culpable conduct, product defect, or

and is offered to prove, for example feasibility, which has been controverted

need for warning

else May be excluded

FRE 403 if probative value is not substantially outweighed by danger of unfair prejudice, etc. and if otherwise admissible under the FREs Admissible

Analytical Chart Text of FREs FRE 407 Subsequent Remedies BARS subsequent remedial measures TO PROVE Negligence, culpable conduct, product defect, or need for warning FRE 408 Compromise BARS Compromise/attempt to compromise a disputed claim and statements/conducted in negotiations TO PROVE liability /invalidity of claim FRE 409 Medical Expenses BARS offers or payment of medical costs TO PROVE liability FRE 410 Pleas BARS (against the ) (1) Guilt plea later withdrawn (2) Nolo contendere plea (3) Statements in plea proceedings (4) Statements in plea talks with prosecution BUT NOT TO PROVE Other things SUCH AS ownership, control, or feasibility, if controverted, or to impeach BUT NOT TO PROVE other things SUCH AS Witnesses bias, lack of undue delay, or obstruction of criminal investigation Also includes when offered in a criminal case and the negotiation related to claim by a govt official

BUT ADMISSIBLE To complete partial account of pleas discussion In perjury prosecution if statement under oath, on record, and in counsels presence

FRE 411 Liability Insurance BARS liability insurance or lack of it TO PROVE negligence / wrongful action

BUT NOT TO PROVE other things SUCH AS agency, ownership, control, or witness bias

Rationales (1) = Rationale Based on Limited Probative Value or Risk of Unfair Prejudice (2) = Rational Based on Other Public Policy FRE 407 Subsequent Remedies FRE 408 Compromise (1) Evidence of remedy is often only (1) Compromise may be motivated by a weak evidence of negligence b/c we desire for peace rather than from any cannot say, b/c the world gets wiser concession of weakness of position. as it gets older, therefore it was foolish before. This is especially true if the settlement amount is small. BUT the inference [of negligence] is a possible one b/c repair tends to [BUT statements made during show consciousness that the situation negotiations may mean more.] called for additional safety-precautions (2) To encourage compromise: (2) To encourage remedies: = the more consistently impressive = the more impressive ground for ground. exclusion. [BUT wouldnt potential s make FRE 409 Medical Expenses remedies anyway? (1) Offer is usually made from human impulses. (2) Admitting evidence would tend to discourage assistance. FRE 410 Pleas FRE 411 Liability Insurance (1) A , although not guilty, might offer (1) (a) It is unlikely that the insured are a plea to avoid the risk of loss after more careless or that the uninsured trial and an even greater penalty. are more careful. (Note the absence of any such argument in the ACN) (b) The jury might otherwise seek deep (insured) pockets or reduce (2) Exclusion will promote plearecoveries to insured plaintiffs. bargaining. = the more important ground for exclusion. (2) Exclusion avoids a windfall for the opponent of an insured party.

FRE 407:

Subsequent Remedial Measures Example, you own an apartment building with stairs. While visiting, someone is holding on to the railing, when the railing comes loose and is injured. A week later, you replace the railing with something much stronger and better. The is not allowed to admit evidence suggesting that the owner took subsequent remedial measures. However, the may admit the evidence to dispute the claim that you do not own the apt bldg. Reasoning for FRE 407 is b/c a change to procedure following an accident is not highly probative of negligence. Sometimes it takes an accident to find a better way to do things. The public policy reasons want to encourage parties to take remedial measures instead of making it better for parties to keep the same practices or facing scrutiny for instituting new policies following an accident. DOES APPLY in strict liability and defective product cases. 3rd Party Repairs The public policy of FRE 407 provides no grounds for excluding evidence of third party repairs. Most third parties will not be dissuaded from making repairs just b/c of those repairs might be offered against someone else. The probative value of most subsequent remedies is that they amount to an admission by the that its previous conduct was unsafe. Some courts have found that evidence of 3rd party repairs, although not barred by FRE 407, has too little probative force to get past FRE 403.

FRE 408:

Compromise and Offers to Compromise Forbids the admission of statements made during settlement negotiations to prove liability of the lack of liability. The FRE does not require exclusion when the evidence is offered for another purpose such as proving the bias or prejudice of a witness. Excludes evidence of conduct or statements made during compromise negotiations Public Policy Reasons. No one would negotiate. Everything would go to trial, and everything said in settlement negotiations would be guarded. FRE 408 applies only in civil cases. This rule does not protect offers to compromise made before a claim of some sort has been made FRE 410 addresses compromises (plea agreements) in criminal cases

FRE 409:

Payment of Medical and Similar Expenses Evidence of furnishing or offering to pay medical expenses is not allowed in at trial. Encouraging settlement of a claim under FRE 408 requires broader protection than encouraging a simple offer to pay medical expenses under FRE 409. Unlike FRE 408, dealing with offers of compromise, FRE 409 applies only to the act of paying or offering to pay and does not extend to conducts or statements

surrounding offers to pay expenses Compromises with 3rd Parties: Note that neither FRE 408 nor FRE 409 limits its exclusionary reach to compromises or payments between the two parties in this lawsuit. Evidence that one of the parties in the suit settled with a 3rd party, if offered to prove liability for or invalidity of the claim, is also barred by the FRE. But remember that such settlements may be admissible to show a witnesss bias. Subject to 403, evidence of prior settlements or offers to compromise is admissible to show that a witness was biased or prejudiced, such as where the witness has received a settlement of her own claim against a party that may predispose her to give testimony favoring that party. Conversely, a witness whose own offer to settle has been rejected by a party may be prejudiced against that party. In some cases, a prior settlement agreement may specifically require the settling party to testify in the instant case (although a settlement agreement cannot legally bind a party to testify in a particular way), a fact that directly bears on bias or prejudice. FRE 410: Inadmissibly of Pleas, Offers of Pleas, Plea Discussions and Related Statements Evidence of criminal pleas cannot be allowed against the , but CAN be allowed to show that the prosecutor has offered to drop or reduce charges. Known to have limited admissibility (1) to complete partial account of pleas discussions (2) in perjury prosecution if statement under oath on record an in counsel presence. FRE 407, 409, and 411 all exclude evidence only when offered to prove certain specified things, while admitting it on any other issue. FRE 410 operates differently. The evidence it addresses is always barred except where specifically permitted. Statements the makes during plea negotiations with the prosecutor may not be used to impeach her should she later testify differently at trial. *Motions to Suppress deals with Constitutional issues (i.e. illegal search and seizure) *Motions in Limine deals with things such as excluding statements at trial and (things that do not deal with Constitutional issues) FRE 411: Liability Insurance Evidence of liability insurance is not admissible upon the issue of whether the person acted negligently or otherwise wrongfully This FRE does not require the exclusion of evidence of insurance against liability when offered for another purpose such as proof of agency, ownership, control, bias or prejudice of a witness Why FRE 411? You dont want jurors deciding who should and should not recover based on whether they have insurance, and guesses as to how much insurance will

award. A. Subsequent Remedial Measures Problem 2.1 A wolf being kept in a residential neighborhood attacked a beagle on the other side of the fence. The next day, the owner of the wolf chained the dog to the fence. A young boy walked by the fence and was attacked by the wolf, receiving lacerations, abrasions and bruises. At trial, the (boy) seeks to admit evidence that following the wolfs attack on the beagle, the chained the wolf. For this , the fact that the chained the wolf is not subsequent, b/c it did not happen after the s accident, but after the accident on the beagle. Evidence should be admitted b/c it shows that the knew that that the wolf had dangerous propensities. For the other (Beagle), the evidence should not be admitted b/c the was taking remedial action subsequent to the attack on the Beagle. Therefore, the evidence of the subsequent tying up of the Beagle should not be admitted into evidence, b/c public policy wants to encourage people to take remedial action. Problem 2.2 and 2.3 A woman whose husband was killed by a defective wood chipper sued the machines maker. Using FRE 407, the court barred the from admitting evidence that the later modified its wood chippers. After it was found that the Army had thirty machines just like the one used by the s husband, the once again sought to have the evidence of remedial measures admitted. By admitting this evidence, you could show that it was not the same machine, but in fact a slightly modified chipper. Introduce the evidence of the altering of the chipper to impeach the subsequent remedial measure to show the jury the miss-impression given by counsel. The jury would get the implication from the statement about the Army having the same machine would mislead the jury. In fact, it was a different (i.e. modified machine). B. Compromise Offers and Payment of Medical Expenses Problem 2.4 Ramada hires an architect to prepare a report of the defects in a building to be used as the basis for settlement. The district court excluded the document under FRE 408. Was this correct? Maybe. The report was not a statement made during settlement but in preparation for settlement. The majority of courts would probably exclude this report from entering trial. C. Liability Insurance Problem 2.5 Claims adjuster took s statement while in the hospital following a fall down her friends stairs. Court found that telling the jury of the statement taken by the adjuster would show that the had insurance. assert that the court erred by not allowing this evidence in (that adjuster was employed by the Andersons insurance company). Once testified, several of her statements varied from those on the taped statements. Here, the evidence should not be allowed in, since there was bias. (i.e. she could have been on painkillers, etc. when she gave the taped statement.) Problem 2.6 and expert witness shared the same insurance company. is trying to show bias. During deposition, s attorney asks the expert witness if he knew who his insurance company was and then if he knew who the defendants insurance company was, to which he replied I wouldnt be surprised if it was the same. The court upheld the judges exclusion of the evidence finding that this has no probative value. Insurance can be allowed in to confirm a dispute of ownership

Problem 2.7 Owner of a daycare is criminally charged with failing to report child abuse. Owner seeks to introduce her insurance policy that covers these claims. She argues that this policy is evidence that she lacked a motive to keep such abuse hidden. Liability is not an issueintroduced for another purposestrong probative value, which is not outweighed by prejudice or bias = most likely allowed in! This example shows the opposite of FRE 411s rationale. The is offering up evidence that she is insured, therefore there is no deep pocket issue. wants to show that she had insurance, and therefore she had not reason not to report the child abuse. The Due Process Clause (right to a fair trial) may override FRE 411 and allow the evidence to come in, b/c the has the right to have a defense. D. Pleas in Criminal Cases Problem 2.8 Prosecutor offers to reduce charges from second-degree murder to first-degree murder. refuses. At trial, seeks to admit evidence of the prosecutors offer as 1) a weakness in the prosecutors case, and 2) as a consciousness of innocence, since a guilty person might have taken the deal. While FRE 410 does not admit s criminal pleas, the FRE does not cover offers made by the prosecutor. Evidence of plea negotiations will not be admitted. A. Character Propensity FRE FRE 404: Character Evidence not admissible to prove Conduct; Exceptions; Other Crimes (a) In general, 404 forbids the introduction of evidence as to a persons character or trait of character if it is offered to prove action in conformity therewith (1) Evidence of pertinent trait of character offered by the accuse or by prosecution OR if evidence of trait of character of the alleged victim of the crimes is offered by an accused and admitted under 404(a)(2) AKA MERCY RULE (2) Subject to limitation of 412, evidence of a pertinent trait of character of the alleged victim of the crime offered by the accuse or by the prosecution to rebut the same or evidence of a character trait of peacefulness of the alleged victim offered in a homicide case to rebut evidence when victim was 1st aggressor AKA MERCY RULE (3) Evidence of the character of a witness, as provided in FREs of 607, 608, 609 (b) Evidence of other crimes, wrongs or acts is NOT admissible to prove the character of a person in order to show action in conformity therewith. It may however be admissible for other purposes, SUCH as (not exhaustive) POP A MIKI: Proof of Motive, Opportunity, Preparation, Plan, Absence of Mistake or accident, Intent, Knowledge, Identity, provided that on request by the accused the prosecution shall provide reasonable notice in advance of trial, or during trial is the court excuses pretrial notice on good cause shown (applies in both civil and criminal cases) In a CIVIL case, character evidence is NEVER admissible especially under circumstantial use FRE 404(a)(1) and (2) is most often raised in self-defense cases to prove that the victim was a violent person, to which the merely responded. Under (2) they could

also bring in their own peacefulness or show that the victim was the 1st aggressor. This can ONLY be raised by the . Once they raise it the prosecution has the open door to inquire about the s character OR the victims peacefulness (rebuttal) Prosecution CANT go through the propensity box against the ; however the can go THROUGH the box to show that they did not commit the crime (inconsistent) Reverse 404(b) where the uses evidence to prove it was not him/her that committed the crime FRE 404(a)(1) Character at Issue use is allowed where the proponents aim is to prove existence of the trait, not to prove any action done in conformity with that trait this is not character propensity evidence under FRE 404(a)(1); normally its the element of the crime (i.e. slander, libel, entrapment, negligent entrustment, custody disputes) A doesnt put his character at issue merely by testifying. Taking the stand places the s CREDIBILITY at issue (as opposed to character)(i.e. the prosecution is limited to impeachment evidence (see FRE 609) rather than substantive character evidence) 404(a)(3) Character of a witness In general, they say that once a witness has offered testimony, the opposing lawyer may use character evidence to attack the witnesss credibility. (cross) The witnesss sponsor may then use character evidence to support her credibility. In contrast, FREs 608 and 609 permit either party to initiate an attack on a witnesss character for truthfulness. After application of FRE 404(a)(1),(2) or (3) then use of FRE 405 to use as the manner of proof to enter in the evidence as opinion or reputation in direct and specific instances on cross examination. However under FRE 404(b) you will not need to follow the specific of FRE 405; can ask about either opinion or reputation and by specific instances of conduct; but even under specific instances asked on cross you ARE NOT allowed to bring in extrinsic evidence The Propensity Box Evidence that a person has a particular character trait generally is not admissible to show that the person acted in conformity with that trait at a particular time. In general, a litigant may not lead the jury on a chain of inferences that goes through the propensity box. Prior Bad Acts to prove bad person/dangerous character TO PROVE ACTION IN CONFORMITY THEREWITH

To prove he did THIS act Routes Around the Box: The chain must go around the box in order to be admissible In such situations, FRE 105 directs the trial judge, at the s request, to deliver a limiting instruction to the jury: When evidence which is admissiblefor one purpose but notfor another purpose is admitted, the court, upon request, shall restrict the evidence to its proper scope and instruct the jury accordingly. The judges instruction should explain in clear terms the permitted, around-the-box purpose for which the evidence of other acts is being admitted. If the judge believes the jury will not abide by the limiting instruction, she may choose to exclude the evidence under FRE 403. Examples: 1) Before any witnesses are called in the case, the state calls the s ex-neighbor to testify that the is hot tempered and has a reputation as violent in the neighborhood. Not admissiblethrough the box. Entered to prove action in conformity therewith. 2) Foster parents who killed baby in the bathtub. State seeks to enter evidence that foster parents were kicked out of the foster system in Michigan after reports of bad acts Not admissible 3) Anatomy of a Murder How many men have you killed? Judge: While I dont really like the question, I dont see how I can exclude it. Prosecutor is obviously going through the box. Could be used for knowledge, but in this case the prosecutor only wanted to show that the murder of wifes lover was an action in conformity therewith. But after going through FRE 404, the judge MUST go through with a 403 test. BUT if you go THROUGH the box, a 403 test is not necessary. This is STRICTLY FORBIDDEN Through the box =INADMISSIBLE (9/3/08 & 9/5/08) - Character Evidence B. Proof of Knowledge Problem 3.1 Two separate computer crimes. One in February and one in March. pled guilty to the latter charge but not the former. Prosecution seeks to enter evidence that the pled guilty to the later theft. seeks to file a pre-trial motion to exclude the evidence. A guilty plea by a is NOT covered by 410only the negotiations. Prosecution could argue that they admitted these acts under 404(b) to prove identity and/or knowledge. If judge lets it in403 balancing test. The defense should then ask for a FRE 403 probative value is substantially outweighed by the danger of unfair prejudice OR a limiting instruction under FRE 105 to consider this evidence only as to whether the had knowledge. The Prosecution is going to win on this argument. This is not being offered to Around the box = ADMISSIBLE (As long as it survives 403)

show propensity, b/c the pool of people that could have committed this crime is so small, that the probative value is not outweighed. Problem 3.2 Davis arrested for drug distribution. No evidence of drug activity found on his person at the time of arrest. State seeks to admit evidence of Davis prior conviction for drug distribution to show that defendant had knowledge of the drug trade, and therefore his identity as the seller in the earlier transaction. Defense argues that the pool of people with knowledge of the drug trade is so large and does not mean my client was the one at the scene of this crime. Bring in 403. State would NOT be allowed to admit the evidence. (At the real trial, district court let the evidence in, but appellate court reversed and granted defendant a new trial) Problem 3.3 a RR brakeman was injured in a train crash caused by the drunken engineer. sues RR Company for negligence. introduced testimony that the engineer was an alcoholic and that the RR knew about it. The court let it in, with a limiting instruction first stating that the did not attempt to prove that engineer was drunk at the time of the evidence, but merely that he had a history of being intoxicated and secondly telling the court you could not look to his general bad reputation at the time, but for the purpose of showing that his reputation was so generally bad that the RR had knowledge/notice of his prior drinking problems and allowed him to continue as operator of the train. (How the instruction should be finished) This evidence should/was allowed in. Defense will want to sever the trial and try the D separately from the RR company. Proof of Motive Problem 3.4 Mr. Peltier killed 2 FBI agents, after being followed. In 1972, (3 years before this crime) Peltier pled not guilty to attempted murder and was released on bond. He did not appear for trial and had an outstanding warrant for his arrest. The governments best argument is to show that this was allowed under 404(b), which showed defendants motive and knowledge. Peltiers best argument is that they are attempting to prove action in conformity therewith and violates FRE 404 and then goes to 403 which would highly prejudice the . Most likely allowed in, at the district court level and on appeal. Problem 3.5 Woman sought to get off a street car and 18 Street. Car did not stop. Then wanted to get off at 19th Street. Car did not stop. Finally the woman got off at 20th Street, but the car did not stop long enough and the woman was injured. At trial woman seeks to admit evidence that the car did not stop long enough at 18th and 19th Streets. The court did not allow it in, and on appeal, the court held that the trial court committed error by not allowing this testimony. The appellate court believed that the prior acts of the failure of the street car to stop at all indicated that the driver was in a hurry, and showed that he was negligent. Evidence allowed in. Proof of Identity Problem 3.6 Return to Peltier case. Two FBI agents were murdered in June of 1975. In November of 1975 Oregon State Police pulled over two vehicles. In the vehicles, the police found Agent Colers revolver, with Peltiers thumbprint. The police also found various bomb making supplies. The police also found Peltiers AR-15, the civilian counterpart of the M-16, a high velocity, small caliber weapon. Doctor had testified that the agents were shot with a high velocity, small caliber weapon. objected to the admission of this evidence, including the weapons. How should the judge have FREd? Peltier had previously claimed that he was not there at the time of the agents murders. Having a revolver with his finger prints on it, goes to identity under 404(b). Other evidence maybe not, but this is definitely allowed in! The several firearms and notes probably NOT

let in! [403] In this instance, the trial court allowed ALL evidence in, but this is one of the issues on appeal. Problem 3.7 Atlanta Police Officers found cocaine, along with lottery numbers and a list of bets at the Atlanta apartment. Occupancy was disputed and the government seeks to admit past charges alleging that Jones (the suspected occupant of the apartment) had been convicted of illegal commercial gambling in 1987. The state then hopes to prove that it was more than likely that Jones was involved in gambling in 1991, and therefore the one that lived in the apartment. This evidence should NOT be allowed in, since the probability of those who gamble is outrageous and cannot be said to show that one who gambled in the past, gambles now AND has cocaine (a totally unrelated and unfounded charge) and is therefore the one that lives in the apartment. This evidence was allowed in at the trial court level, but reversed on appeal. (This type of problem would be a good essay questioncould go either way). Problem 3.8 Police found bicycling brochures in an apartment, along with cocaine and packaging supplies. The state then seeks to admit evidence that the was an avid bicycle racer three years ago. This evidence is admissible since it is not a bad character act. BUT this would also depend on what other evidence the state had, since a brochure of bicycle racing next to cocaine, without fingerprints or something stronger linking the defendant is ridiculous and too far of a stretch since anyone could leave unrelated material next to cocaine in the hopes that it will incriminate someone else. The rationale is that the jury is not going to punish you for being a bike rider. Unlike the problem above, the jury would be more likely to punish an illegal gambler. This evidence would most likely be allowed in. Modus Operandi (M.O.) used more for IDENTITY One way to prove guilt when identity is in dispute is to show that the crime matches the s M.O. If we know the committed a particular crime in the past, and the present offense matches that crime in idiosyncratic ways, we may infer that the committed the present offense as well. Not the idea, that this is the s kind of crime, but the idea that this could not be anyone elses crime. (Nobody else could have done this, b/c the crime is so unique). If not dealing with an identity issue, then dont even deal with M.O. For this route around the propensity box to be legitimate, the similarities between the two crimes must be so distinctive that the inference that nobody else could have committed this crime overcomes the jurys temptation to engage in propensity reasoning pure and simple. Narrative Integrity (Res Gestae) Bring in the evidence b/c it is apart of the story and without that evidence it would not make sense. Commonly used rationale to go around the propensity box (use of witness testimony) along with POP A MIKI exceptions Problem 3.10 The playing Russian roulette with the witness four years earlier. is charged with possession of a firearm with an obliterated serial number. The propensity shows that he owned the gun before and that he would possess a gun with the crime charged. (RIGHT THROUGH THE BOX) Under 404(b) would want to use for identity based on testimony of the witness; (narrative integrity) b/c he was playing Russian roulette with her life

Absence of Accident stating that it was an accident the first time the incident occurred, then it happens a second time the says its an accident again. The prosecution can try to enter in evidence to prove it was not an accident the second time. Problem 3.11 was charged with shooting his wife, after a bullet accidentally discharged while he was cleaning it. State seeks to admit evidence that three years earlier, killed his first wife after the gun accidentally discharged. Prosecutor would argue that twice, makes it less likely that it was an accident. Defense would argue that prosecutor is simply taking a trip THROUGH the box. Prosecutor could also proffer the previous event to prove knowledge, since obviously had experience with guns and you would think after killing someone (HIS WIFE!!) he would never go near a gun againor at least take precautions. Introduce to show POP A MIKI. Problem 3.12 The is on trial for throwing a dog out into the street. The dog was killed by a car. The Prosecution seeks to admit evidence that the previously killed another dog by clubbing it to death when he was in the Navy. The judge has tentatively FREd that he will allow testimony from the s former navy supervisor who said that the admitted that he beat a mutt to death. Was the judges ruling right? Previous behavior may go to knowledge. However, evidence should possible be kept out b/c evidence is not admissible to show action in conformity therewith. The judge allowed the evidence in order to rebut the fact that it was an accident. Most judges will probably NOT let this in b/c it goes straight through the propensity box. Doctrine of Chances The initial decision the facing the trier of fact is whether the uncharged incidents are so numerous that it is objectively improbable that so many accidents would befall the accused. Rex v. Smith was charges with murdering Bessie Mundy. They married after she had inherited 2500 pounds from her father. He took his wife and sent for the doctor a couple of times, saying that his wife was having a fit. The doctor came and they both went upstairs to find her head up in the bath tub. wept to his landlord, and made a statement that, Wasnt it a jolly good job that I got her to make her will. The Prosecution sought to admit evidence that that two prior women that the was married to died in baths. The judge admitted the evidence only for the purpose of helping the jury to draw an inference as to whether the death of Ms. Mundy was accidental or designed by the prisoner. The jury found the guilty. FRE 404(b) specifically lists proof of absence of mistake or accident as one of the other purposes for which evidence of other acts may be used. The sheer improbability that the s three wives could have all died in a bath without foul play has become known as the doctrine of chances, recognized in some quarters as an around-the-box use of other-acts evidence. The Court of Criminal Appeals upheld the conviction. The Huddleston Standard (Huddleston v. United States) Is there a conditional relevance problem? FER 104(b) EXAM question: 104(a) the judge determines by a preponderance of the evidence 104(b) the court determines if the jury can determine by the preponderance of the evidence You can only get in evidence of a signature crime (same MO) when identity is an issue. In order to get evidence in, the state would have to show the judge that a jury could find by a preponderance of the evidence that the defendant is the person who committed the prior act.

(104(b)) (9/8/08 & 9/10/08) - Character Evidence contd Past Acquittal Problem 3.13 commits a bank robbery. The state (USVI) then calls a woman to the stand to testify that the and another man robbed her home. In a previous trial, the had been acquitted of the home robbery. Should the state be allowed to bring in this testimony, since the has already been acquitted and his identity doubted? In the first trial, the jury had to prove guilt beyond a reasonable doubt. However, to admit this evidence in the bank robbery trial, the state need only prove (at pretrial hearing) a preponderance of the evidence. The State is not prevented from admitting prior acts (even those that are not convictions) to prove identity. Ask for a limiting instruction under FRE 105. Although there is probably great prejudice, and the evidence most likely would not be allowed in under 403, it is up to the trial courts discretion. Evidence allowed in. D. Propensity Evidence in Sexual Assault Cases True exceptions to the propensity evidence ban FER 404(b) FRE 413: Evidence of similar crimes in Sexual Assault Cases (i.e. other sex crimes) (a) In a criminal case in which the is accused of an offense of sexual assault, evidence of the s COMMISSION of another offense or offenses of sexual assault is admissible and may considered for its bearing on any matter to which it is relevant. (b) In a case in which the govt intends to offer evidence under this rule, the govt attorney shall disclose to the , including statements of witnesses or a summary of the substance of any testimony that is expected to be offered, as least 15 days before the scheduled dare of trial or at such later time as the court may allow for good cause (c) This rule shall not be construed to limit the admission or consideration of evidence under any other FRE (d) For purposes of this rule and FRE 415, offense of sexual assault means a crime under federal law or the law of a state (as defined in U.S.C 18 513) In a criminal case where is CONVICTED of sexual assault, evidence of prior crimes of sexual assault, including sodomy, assault and molestation, may be admitted for consideration for its bearing on any matter to which it is relevant. If there is an ACQUITTAL OR ARREST and NO CONVICTION then there is a Huddleston issue. Thus applying the idea of FER 104(b) where the judge determines if the jury will decide this. (Conditional Relevance) FRE 414: Evidence of similar crimes in Child Molestation Cases (a) In a criminal case in which the is accused of an offense of child molestation, evidence of the s COMMISSION of another offense or offenses of child molestation is admissible and may considered for its bearing on any matter to which it is relevant. (b) In a case in which the govt intends to offer evidence under this rule, the govt attorney shall disclose to the , including statements of witnesses or a summary of the substance of any testimony that is expected to be offered, as least 15 days before

the scheduled dare of trial or at such later time as the court may allow for good cause (c) This rule shall not be construed to limit the admission or consideration of evidence under any other FRE (d) For purposes of this rule and FER 415, offense of child molestation means a crime under federal law or the law of a state (as defined in U.S.C 18 513) FRE 415: Evidence of similar acts in CIVIL CASES concerning sexual assault OR child molestation cases (a) In a civil case in which a claim for damages or other relief is predicated on a partys alleged COMMISSION of conduct constituting an offense of sexual assault OR child molestation, evidence of that partys commission of another offense or offenses of sexual assault OR child molestation is admissible and may considered as provided in FER 413 and FER 414 of these rules. (b) A party who intend to offer evidence under this rule shall disclose the evidence to the party against whom it will be offered, including statements of witnesses or a summary of the substance of any testimony that is expected to be offered, as least 15 days before the scheduled dare of trial or at such later time as the court may allow for good cause (c) This rule shall not be construed to limit the admission or consideration of evidence under any other FRE Courts apply the FRE 401 relevancy (beginning of flowchart) and FER 403 balancing test of probative value vs. substantially unfair prejudice to the after looking at FER 413 and FER 414 (good to look at flow chart on pg. 151) Notes on FER 413 415: By enacting them in 1994, Congress sought to ensure that federal trial judges could admit evidence of past sexual misconduct in sexual assault and child molestation cases without having to stretch the meaning of intent, motive, and plan. FER 405(a) does NOT apply to FER 413 FER 415 b/c they focus on specific acts/instances. Whereas, 405(a) looks at ALL history of the Problem 3.14 is jealous of his wife. He controls her using non-lethal violence. During estrangement, the wife is murdered. If the Prosecutor offers evidence of past battering incidents under Readers theory, how will you argue against admission? How should the judge FRE? The evidence should not be allowed in b/c it goes straight through the propensity box. No overall scheme was shown to show that the killed his wife, the evidence would NOT be allowed in. E. Proof of the Defendants and Victims Character FRE 404: FRE 405: Character at Issue (see above) Methods of Proving Character (Manner of Proof) (a) Reputation or Opinion. In all cases in which evidence of character or a trait of character of a person is admissible, proof may be made by testimony as to reputation or by testimony in the form of an opinion. Can be brought in on direct but on

cross-examination inquiry is allowable into relevant specific instances of conduct. (b) Specific instances. In cases in which character or a trait of character of a person is an essential element of a charge, claim or defense, proof may also be made of specific instances of that persons conduct. (i.e. may only introduce specific incidents if the trait they are trying to prove is essential to the charge)(can ONLY be done on cross unless if not brought in by character evidence, which is 404(b)) Only when the existence of the character at issue (essential element) and not any conduct done in conformity with that trait is the thing to be proved. APPLIES in cases such as rebutting an embezzlement, entrapment defense, rebutting a defense to truth in a libel, defamation OR slander case and resolving parental custody disputes (Use of FRE 404(a)(1) then FRE 405 for this instance; NOT limited on which way to use the evidence) CHARACTER IS NOT ADMISSIBLE IN CIVIL CASES 405(a) provides that whenever proof of character is allowed under either FRE 404(a)(1) or FRE 404(a)(2) (the exceptions), that proof may take the form of either reputation or opinion evidence (direct) and specific conduct (brought in on cross). However if it is NOT brought in on character then FRE 405 DOES NOT limit the form of evidence, can use either opinion/reputation or specific conduct in direct and/or cross) Under FER 405: FER 404(a)(1) permits criminal s to offer proof of pertinent traits of their own character. FER 404(a)(2) allows permits criminal s to offer proof of pertinent traits of the alleged victims character. Both FREs give the the SOLE OPTION to open the character inquiry. The only EXCEPTION to this general FRE is a narrow one: In HOMICIDE CASES FER 404(a)(2) permits the prosecution to offer evidence of the victims peaceable character if the has offered evidence (even non-character evidence) that the victim was the 1st aggressor. Under cross-exam is limited to the inquiry of the witness; she MAY NOT introduce extrinsic evidence of the specific instance prosecution is stuck with the answer Limitations are based on doing a voir dire (no jury) then the judge and the attorneys choose which evidence will come in Problem 3.15 charged with committing a drive by shooting. At trial, the defense counsel asks if you had to do this over again, would you shoot Mr. Cusick, or would you shoot out of the car? to which she replied I wouldnt shoot anybody. Then the Prosecution asks, Youve shot at other people before havent you? If defense counsel objects to this questioning, how should the court FRE? The court should admit the evidence, since she has in effect become her own character witness to which the prosecutor is allowed to cross-examine as to specific incidents of conduct. Also consider impeachment proceedings b/c a prior inconsistent statement to detectives. Problem 3.16 K is charged with shooting and paralyzing R. K claims self-defense. During the trial,

R runs into Ks brother in the hallway and shouts, there is his brotherremember his face. The defense seeks to admit this evidence as an incident of Rs violent nature. If prosecutor objects, how should court FRE? Judge should not allow this evidence in since it is a specific incident offered by the defense. The defense could only call a witness to testify as to the s aggressive behavior. Problem 3.17 Recall testimony from U.S. v. James, in which the testified that Ogden/boyfriend had told her about people he had stabbed and beaten. If the Prosecutor had objected to this testimony as improper character evidence, how should the court have FREd? Objection as to whether past specific acts of violence should be admitted. Whether she can offer this character evidence of to bolster claim of self-defense b/c he had been violent in the past? Is the offering evidence to show victims propensity to be violent. Goes to state of mind what she believes he is capable of. Does not go to his character trait does not matter if he did all those things; matters if she thought he is a violent person. Evidence is not being offered as his character trait; being offered to show state of mind when she gave daughter gun gets around propensity box. Problem 3.18 The was charged with murder and the claimed that he acted in self-defense. The jury found him guilty. On appeal, he complains that the court wrongly excluded the expert testimony of a toxicologist. The toxicologist would testify that morphine, cocaine, heroin, and alcohol were found in the Victims body. He said that the cocaine causes violence. Was the trial court right to exclude this evidence of the Victims character? claimed self defense. Doctor testified victim had cocaine, morphine and alcohol in his body to prolong effects of cocaine increases likelihood that victim was aggressor. wants to offer victims character evidence of propensity to be violent. Does not go to character evidence. s state of mind is at issue, not the Vs state of mind. Evidence goes to show victim was affected by drugs at time incident occurred. The toxicologist can testify as to his expert opinion as to what these drugs to when they mix, but cannot testify as to specific acts/character. Prosecution best argument is 403 overly prejudicial to the victim but probably will lose. F. Evidence of Habit FRE 406: Evidence of Habit Evidence of the habit of a person or of the routine practice of an organization, whether corroborated or not and regardless of the presence of eyewitnesses, is relevant to prove that the conduct of the person or organization on a particular occasion was in conformity with the habit or routine practice. Key words are predictable and predictive conduct. Test: Whether the conduct is virtually automatic and is repeated several times in the past. It goes around the box unconscious behavior. Problem 3.19 Doctor sued for the death of a patient when what patient believed to be antihistamines were in fact steroids which killed the patient. The patients estate sought to introduce evidence that the doctor had prescribed steroids to eight other patients under the belief that they were antihistamines. Allowing this evidence in as a habit would depend on the number of patients treated, the length of time from which the steroids were prescribed, etc. Most likely inadmissible, b/c the Doctor had a great number of patients (over 20 years) and only eight were said to have been prescribed steroids. Essay answer would be that it is not enough that he did this to 8 people over 20

years and therefore; the probative value does not outweigh the prejudicial value. (9/16/08 & 9/18/08) - Character of Truthfulness Route of Admissibility (Impeachment Evidence) Non-character evidence of statements or facts From other flow chart up top FRE 401 & FRE 402 FRE 608(a) Evidence in form of opinion or reputations FRE 608(b) Or, on cross only, evidence of specific acts FRE 609 Evidence of certain certain past convictions past inconsistent conflicting statements evidence/ circumstances bias

If offered to prove character for untruthfulness

If offered to prove a specific lie

Or offered to prove character for truthfulness after attack on same FRE 403 Except past convictions admitted under FRE 609(a)(2) possibly admissible Final Exception to the Ban on Character Evidence: FRE 404(a)(3) (see above) Introducing evidence of a witnesss character for truthfulness is ONE way to impeach. Also: (BCLMP) NON-CHARACTER EVIDENCE 1. Evidence of prior inconsistencies 2. Motive 3. Bias (non-character method of impeachment) 4. Contradictionwitness doesnt have the facts right or is mistaken * most common 5. Lack of memory or ability to perceive (defect in sensory or mental capacity) You DONT have to use FRE 608 in order to use the non-character based impeachment (can use in direct or cross examination; may ALSO bring in EXTRINSIC EVIDENCE) FRE 607: Who May Impeach The credibility of a witness may be attacked by any party, including the party calling

else Inadmissible

the witness. FRE 608: Evidence of Character and Conduct of Witnesses (a) The credibility of a witness may be attacked or supported by evidence in the form of opinion or reputation, but subject to these limitations. (1) The evidence may refer only to character for truthfulness or untruthfulness (not to character for peaceableness, temperance, or anything else); and (2) Evidence of truthful character is admissible only AFTER the character of the witness for truthfulness has BEEN ATTACKED by opinion or reputation evidence or otherwise. (b) Specific instances of the conduct of a witness, for the purpose of attacking or supporting the witness character for truthfulness, other than conviction of crimes as provided in 609, may not be proved by extrinsic evidence. They may, however in the discretion of the court, if probative or truthfulness or untruthfulness, be inquired into ONLY on cross-examination of the witness (1) concerning the witness character for truthfulness or untruthfulness, or (2) concerning the character for truthfulness or untruthfulness of another witness as to which character the witness being cross-examined has testified The giving of testimony, whether by the accuse or by any other witness, does not operate as a waiver of the accuseds or the witness privilege against selfincrimination when examined with respect to matters that relate only to character for truthfulness This FREs applies to both CRIMINAL and CIVIL cases Under 608(a) You cannot bolster your witness, until they have been attacked! Under 608(b) You can only question a witness about specific instances of conduct; if they dont answer under oath to the truth you CAN NOT bring in evidence to show other wise. IF it is a conviction under 609 and you are NOT allowed to bring it in, you CAN NOT use FRE 608 to bring it in FRE 608(b) allows more liberal use of specific instances of conduct than does FRE 405(a), which governs the forms of character evidence offered under FRE 404(a)(1) and 404(a)(2). That is, FRE 405(a) permits inquiry into specific instances of conduct ONLY during cross-examination of the character witness Does the question relate to the witnesss character for truthfulness or untruthfulness? If YES, continue analysis; If NO, FRE 608 does not apply Was he asked Cannot rehabilitate a witnesss character for truthfulness until it has been attacked Methods of proving character for truthfulness? Opinion or reputation testimony FRE 608(a); Questions showing untruthful character by specific acts only on cross FRE 608(b) (NO extrinsic evidence is permitted under FRE 608) Evidence of a past conviction of a witness can be shown by extrinsic acts FRE 609 Evidence must survive the FRE 403 balancing test Six Limitations on 608(b)(1):

There are 2 explicit limitations: The specific instance of conduct MUST be probative of truthfulness or untruthfulness and it may not be proved by extrinsic evidence. The ban on extrinsic evidence means the lawyer must accept the witnesss answer to the question. (i.e. If the lawyer asks about a specific lie allegedly told by the witness at some point in the past and the witness denies it, that is the end of the matter. The lawyer may not call a second witness to prove that the first witness indeed told the lie in question. 3rd limitation is that the inquiry must survive a FRE 403 weighing test if using FRE 608(b) for the discretion of the court 4th limitation is the lawyer must have in his possession information that reasonably leads him to believe that the acts of conduct have in fact been committed. 5th limitation on cross-examination under 608(b)(1) is imposed by FRE 611(a)(3): The trial judge must exercise reasonable control to protect witnesses from harassment or undue embarrassment. 6th limitation prescribes that most courts will not permit a lawyer to use FRE 608(b) to offer evidence that is of a type covered by FRE 609, but is excluded by 609. (i.e. if a lawyer tried to use an 11 year old conviction, excluded by FRE 609(b), as a specific instance of conduct under FRE 608(b), most courts would exclude the evidence) FRE 608(b) is NOT a back door for admission of evidence not admissible under FRE 609. Example Who can ask? Assume that a witness testifies to an accused murders reputation for peaceableness under FRE 404(a)(1). During cross-examination, the prosecutor could properly ask whether the witness has heard about a previous incident in which the defendant was violent (FRE 404(a)). If the now takes the stand, FRE 405(a) would NOT permit the prosecutor to crossexamine him about the previous incident. There are, however, two situations in which the Prosecutor might be permitted to question the about the old incident. 1st the prosecutor could argue that she is offering the evidence for a non-propensity purpose under 404(b). 2nd if the has vouched for his own character (for example, by saying he is a non-violent person), then the arguably has made himself his own character witness, and the question may be proper under FRE 405(a). Problem 4.1 L sues P for injuries that occurred in a bar brawl. L brings witness R who states that she saw P strike L with a pool stick. This is a civil trial. L then calls Rs neighbor to testify that: (1) Rs reputation in the neighborhood for peaceableness is good; and (2) Rs reputation in the neighborhood for truthfulness and veracity is good. 1) Inadmissible. L has offered testimony of Rs peaceableness. No such evidence is admissible under 404(a)(2) b/c, first this is a civil case, and second, R is neither the accused nor a victim. Such evidence is not admissible under 608(a)(1), b/c it does not concern Rs character for truthfulness or untruthfulness. 2) Inadmissible. Rs character for truthfulness has not yet been attacked. P calls a co-worker of R to testify that: (3) R sold a faulty car, she told me the brakes had been replaced, but she was lying; and (4) My opinion, based on all my dealing with R is that she is a liar. (3) Inadmissible. P has attempted to impeach R by asking about a lie she once told. (i.e. P is offering evidence of a specific instance of Rs untruthful conduct and is doing it direct examination of a co-worker) 608(b) permits inquiry about specific instances of conduct, but ONLY on cross-examination. (4) Admissible. Here the co-worker offers her opinion that R is a liar (shorthand way of saying that R has a bad character for truthfulness). On cross examination of the co-worker, L asks: (5) Is it true that you were expelled from college for a semester after trashing your dorm room in a drunken rage; and (6) Did you lie on your med-school application about whether you had disciplinary offenses during

college? (5) Inadmissible. L has attempted to impeach the co-worker by asking, on crossexamination, about the drunken rage. Although 608(b)(1) permits injury about specific acts, those acts must be probative of truthfulness or untruthfulness. This incident does not qualify. (6) Admissible. L has attempted to impeach the co-worker by asking, on cross-examination, about a lie told in a semi-official context. Although the underlying dorm incident did not reflect on the coworkers character for truthfulness, her later lie about the same incident does. Problem 4.2 was a prominent lawyer before working at Tyco. Prosecutors want to go into detail about when he lied on his application for a Utah drivers license that it has not been revoked in 10 years and in NY it was revoked. That he lied to get of jury duty in NY stating that he lived in Utah but that did not happen until a year later. The third incident of indiscretion was by writing a letter for the presidents daughter to get in Columbia Business School telling them she had worked for Tyco steadily or 2 years. The trial judge ruled that if he testified, then they could cross-examine him on it. I do not think it is the right ruling, b/c under FRE 608(a) and (b) if the brought in a witness to say he was a good guy, then they should cross exam them to see if they knew of these allegations. Also under FRE 405(b) IF the crime charged has the element of the crime being of honesty then the prosecution should be able to bring the evidence of him doing such allegations through their own witnesses; thus the can cross examine them if the choose to. Use of Past Convictions Focus on 609 FRE 609: Impeachment by evidence of conviction of crime (a) For the purpose of attacking the character for truthfulness (credibility) of a witness. (1) Evidence that a witness other than an accused has been CONVICTED of a crime SHALL be admitted, subject to FRE 403 if the crime was punishable by death or imprisonment in excess of 1 year under the law under which the witness was convicted, and evidence that an accused has been convicted of such a crime shall be admitted IF the court determines that the probative value of admitting this evidence outweighs the prejudicial effect. (2) Evidence that any witness has been CONVICTED of a crime SHALL be admitted if it involved dishonesty or false statement, REGARDLESS of punishment. (b) Time Limit. CONVICTIONS are NOT ADMISSIBLE if a period of more than 10 years has elapsed since the date of the conviction or of the release of the witness from the confinement, whichever is the LATER date, UNLESS the court determines in the interest of justice, that the probative value of the conviction supported by specific facts and circumstances substantially outweighs its prejudicial effect. (FRE 403) Evidence of a conviction older than 10 years is not admitted unless the moving party gives sufficient advance written notice of intent to use as to give the adverse party the opportunity to contest such evidence. (c) Effect of pardon, annulment, or certificate of rehabilitation. Evidence of a convictions is NOT admissible under this FRE if (1) the conviction has been the subject of a pardon, annulment, certificate of rehabilitation, or other equivalent procedure based on a finding of the rehabilitation of the person convicted, and that the person has NOT been convicted of a SUBSEQUENT crime that was

punishable by death or imprisonment in excess of one year, or (2) the conviction had been the subject to a pardon, annulment or other equivalent procedure based on a finding of innocence (NOT GUILTY) (d) Juvenile adjudication. Evidence of a juvenile adjudication is generally NOT admissible under this FRE. The court MAY in a criminal case allow evidence of a juvenile adjudication of a witness other than the accused if the conviction of the offense would be admissible to attack the credibility of an adult and the court satisfied the admission in evidence is necessary for a fair determination of the issue of guilt or innocence (e) Pendency of appeal. The pendency of an appeal thereform DOES NOT render evidence of a CONVICTION inadmissible. Evidence of the pendency of an appeal is ADMISSIBLE Under FRE 609(a) the time of punishment is what is usually allotted to the particular crime, regardless of how long the particular defendant servesno time off for good behavior! Under FRE 609(a)(1) and (2) speak about crimes that are based on untruthfulness such as perjury, false pretense, false statement, criminal fraud, embezzlement or any crimen falsi type of crime which has the element of deceit, untruthfulness or falsification Under FRE 609(c) a persons successful rehabilitation DIMINISHES the probativeness of past crimes, and of course a later finding of innocence would reduce probativeness to near ZERO Under FRE 609(a)(1) Evidence offered against the ACCUSED in a criminal case must survive a STRICTER weighing test: It is admitted ONLY IF its probative value outweighs its potential to cause unfair prejudice to the . The Brewer court laid out the 5 factors that generally govern this weighing test 5 factors: (1) The nature of the crime (2) The time between the conviction and the witnesses subsequent history (3) Similarity between the past crime and the charged crime (4) Importance of s testimony; and (5) The centrality of the credibility issue (NOT NEEDED FOR WITNESSES) Under 609(d) the previous crime CANNOT be admitted in a civil case or against the criminal In some cases, the court will allow under the FRE 403 to make the conviction admissible HOWEVER the attorneys are ONLY to refer to it as a felony conviction (basically stripping the identity of the crime) Generally permits opposing counsel to impeach a witness with evidence of their past crimes; however under circumstances where the conviction is older than 10 yrs and its a crime of crimen falsi (dishonesty), the court states that the FRE 609(b) will trump and make the conviction inadmissible

Is there mention of a conviction? If no mention, analyze under FRE 608 If yes, analyze under FRE 609 If a conviction is Inadmissible under FRE 609, can it still be admitted under FRE 608? You could bring it under FRE 608 but the advisory committee says you should not be using another rule if there is a rule relating to that subject FRE 609 in Force: Problem 4.2 took the stand and testified that he was hundreds of miles away at the time of the crime. The prosecution then attempted each of the following modes of impeachment: (1) She asked the on cross-examination whether he shot a man in the arm during a drunken barroom brawl. (2) She called a witness to testify that he saw the shot a man in the arm during a drunken barroom brawl. (3) She offered evidence that the was convicted of assault and battery by a dangerous weapon and sentenced to 5 years in prison for having shot a man in a drunken barroom brawl. (4) She asked the on cross-examination whether he was convicted of turnstile-jumping and sentenced to 3 months in county jail. (5) She offered evidence that the was convicted of lying to a federal investigator and sentenced to 2 years in federal penitentiary. ANSWERS: (1) Inadmissible (2) Inadmissible (3) Maybe admissible (4) Probably inadmissible (5) Possibly admissible Problem 4.4 appealed his conviction of distributing cocaine. contends that the court should not have allowed in a prior conviction of selling a controlled substance for impeachment purposes. The govt offered the prior conviction pursuant to 404(b). The testified in his own defense and on cross, the govt offered the s prior conviction. The prior conviction was admitted. argues that 609(a)s balancing test is stricter. argues that after finding that the prejudice of the prior conviction substantially outweighed the probative value under 404(b), and therefore the court abused its discretion in admitting the same evidence under 609, where it would be excluded if the prejudice merely outweighed its probative effect. Is right? If the did not take the stand, then the evidence would not come in. Under 609, it should come in if the court did the proper balancing test. Just b/c it did not come in under 404, it does not mean it will not come in under 609. Also, if evidence is excluded under one FRE, it does not mean it will be excluded under another. For anything other than proving character for truthfulness, extrinsic evidence is allowable. Problem 4.5 In a suit against his current company for age discrimination, the company asks the if he was convicted of tampering with electric meters while helping McDonalds franchises to lower their electric bills. Should this evidence be allowed in? It was admitted b/c it involved dishonesty. His purpose was to deceive the electric company by tampering with their meter and fell into the category crimen falsi. FRE 609(a)(1) and 609(d) therefore extend greater protection to criminal who take the stand than to civil parties or ordinary witnesses. FRE 609(a)(1) establishes a stricterthan-usual weighing test that a prosecutor must overcome before offering evidence of past convictions against a criminal . FRE 609(d) altogether bars evidence of a s juvenile adjudications. FRE 609: Case Notes on Appellate Review FRE 609 is especially protective of criminal s. If the trial judge disregards the FREs safeguards and wrongly admits evidence of past convictions to impeach a , the normally may appeal. The Supreme Court has ruled, however, that a MAY NOT appeal from the trial judges ruling unless 2 conditions are met: (1) The MUST in fact testify at trial (2) The prosecutor must introduce evidence of the contested conviction

Rehabilitation Problem 4.6 The is standing trial for a bank robbery. After the prosecution rests, the calls Louise Johnson as an alibi witness. On direct examination by defense counsel, Johnson testifies that at the time of the bank robbery, she and the were having a late lunch. Consider the following: (1) On cross-examination of Johnson, the Prosecutor asks, Arent you in business with the s father? And Johnson answers yes. May the now call a witness to testify that Johnsons reputation for truthfulness and veracity is good? (2) On cross-examination of Johnson, the prosecutor asks, Isnt it true that on the day of the bank robbery, the deli was closed for a religious holiday? Johnson answers, Thats right, I forgot. We had to eat next door instead. May the now call a witness to testify that Johnsons reputation for truthfulness and veracity is good? (3) On cross-examination of Johnson, the prosecutor asks, Didnt you state in an application for a bank loan that you had no outstanding debt, when in fact you still owed $82K in student loans? Johnson answers, Yes. May the now call a witness to testify that Johnsons reputation for truthfulness and veracity is good? (4) After Johnson completes her testimony, the prosecutor offers evidence that, 6 years earlier, she was convicted of armed robbery and sentenced to serve 4 years in state prison. The then call Johnsons neighbor, who testifies that he has known Johnson for 8 years and that, in his opinion, she is a truthful person. On cross-examination of the neighbor, may the prosecutor ask, Do you know that Ms. Johnson stated in an application for a bank loan that she had no outstanding debt, when in fact she still owed $82K in student loans? ANSWER: (1) INADMISSIBLE (2) PROBABLY INADMISSIBLE (3) ADMISSIBLE (4) ADMISSIBLE Rehabilitating the Credibility of a Witness: Rehabilitation concerns a partys attempts to SUPPORT a wittinesss character for truthfulness. FRE 608(a)(2) makes clear that one party may rehabilitate its own witnesss character for truthfulness only after the other party has attacked the witnesss character for truthfulness. Any of the forms of attack provided in 608 and 609 clearly qualifies. There is a definite attack when: (1) the other party has offered opinion or reputation testimony of the witnesss bad character for truthfulness (2) the other party has elicited on cross examination of specific acts of the witness that are probative of untruthful character or (3) other party offered evidence of a past conviction of the witness under 609 Therefore, if one of the parties has used any of the three attacks above, then the other party may use ANY techniques permitted in FRE 608 to rehabilitate the witnesss character for truthfulness. These 3 ways MAY NOT construed as attacks on the truthfulness of a witnesss testimony in this proceeding: evidence of bias (no), contradiction by past inconsistent statement (maybe), and contradiction by conflicting evidence. Under the 2003 amendment, the ACN stated that extrinsic evidence can be admissible when using one of these 3 ways that were mentioned earlier. Problem 4.7 Use of Extrinsic Evidence Problem 4.8 (1) On cross-examination of Johnson, the s alibi witness, the prosecutor asks, Didnt you state on an application for a bank loan that you had no outstanding debt, when in fact you still owed $82,000 in student loans? Johnson answers, No. May the prosecutor now call the banks loan officer to testify that Johnson indeed claimed she had no outstanding debt as well as a record-keeper from the student loan agency to testify about the status of her student loans? (2) On

cross-examination of Johnson, the prosecutor asks, Arent you in business with the defendants father? Johnson answers, No. May the prosecutor now call a witness to testify that Johnson is in business with the s father? ANSWERS (1) Inadmissible (2) Admissible Extrinsic Evidence There are two points in the FREs governing character evidence at which the litigant reaches a dead end: (1) Under FRE 405(a), the litigant may ask a character witness on cross-examination whether that witness has heard of a specific act committed by the person about whose character the witness is testifying. Regardless of the witnesss answer, the lawyer may present no other evidence (i.e. no extrinsic evidence) regarding the act. (2) Under FRE 608(b), the litigant may cross-examine a witness about specific instances of conduct that bear on character for truthfulness. But the FRE explicitly states that such conduct, other than the conviction of a crime as provided in FRE 609, may not be proved by extrinsic evidence. If the witness denies having done (or heard of) the specific act, the lawyer may present NO other evidence about it. Problem 4.9 (9/22/08 & 9/24/08) The Rape Shield Law & Competency of Witnesses FRE 412: Sex Offense Cases, Relevance of Alleged Victims Past Sexual Behavior or Alleged Sexual Predisposition (a) Evidence generally inadmissible. The following evidence is NOT admissible in any civil or criminal proceeding involving alleged sexual misconduct except as provided in subdivisions (b) and (c). (1) Evidence offered to prove that an alleged victim engaged in other sexual behavior. (2) Evidence offered to prove any alleged victims sexual predisposition. (b) Exceptions (1) In a criminal case, the following evidence is admissible, if otherwise admissible under these rules: (a) Evidence of specific instances of sexual behavior by the alleged victim offered to prove that a person other than the accused was the source of semen, injury or other physical evidence. (ANOTHER PERSON) (b) Evidence of specific instances of sexual behavior by the alleged victim with respect to the person accused of the sexual misconduct offered by the accused to prove consent or by the prosecution and (THE ) (c) Evidence the exclusion of which would violate the constitutional rights of the . (2) In a civil case, evidence offered to prove the sexual behavior or sexual disposition of any alleged victim is admissible if it is otherwise admissible under these rules and its probative value substantially outweighs the danger of harm to any victim and of unfair prejudice to any party (FRE 403). Evidence of an alleged victims reputation is admissible ONLY if it has been placed in controversy by the alleged victim. (c) Procedure to determine admissibility

(1) A party intending to offer evidence under subdivision (b) must (a) File a written motion at least 14 days before trial specifically describing the evidence and stating the purpose for which it is offered unless the court, for good cause requires a different time for filing or permits filing during trial; and (b) Serve the motion on all parties and notify the alleged victim, or when appropriate, the alleged victims guardian or representative (c) Before admitting evidence under this rule the court must conduct a hearing in camera and afford the victim and parties a right to attend and be heard. The motion, related papers, and the record of the hearing must be sealed and remain under seal unless the court orders otherwise Under this rule, in order to bring in evidence under subsection (b) you MUST satisfy subsection (c) The rationale behind this rule is to protect the victim from potential embarrassment, invasion of privacy and sexual stereotyping based on the public disclosure of intimate sexual details Dreams and fantasies fall under the sexual behavior definition of 412(b)(1)(b) 3rd party witnesses ARE NOT protected by this rule, but are offered protection under FRE 404, FRE 608 and FRE 403 In defamation, libel or slander actions that involve statements concerning sexual misconduct in which the evidence is offered to show that the alleged defamatory statements were true or did not damage the s reputation, neither FRE 404 or FRE 412 operate to bar evidence, the controlling is FRE 401 and FRE 403 The Law in Force Past Sexual Behavior with the Accused Problem 5.1 and engaged in sadomasochistic acts, after which she charged the with rape. tries to admit emails in which the wrote of her involvement in other sadomasochistic acts. Under 412(b)(1)(B), the evidence would be admitted as a prior act with the even though it was only by email. However making sure that they provided the information based on subsection (c) of FRE 412. The Prosecution could argue to exclude under 412(a)(2) which makes evidence of the s prior sexual disposition. The court allowed this evidence to come in, b/c the emails are going to show consent. Judge would have to find by a preponderance of the evidence that the emails constituted sexual behavior (Huddleston standard). Judge would determine under 104(b) conditional relevance. Explaining the Source of Physical Evidence Problem 5.2 The , who was charged with rape, worked as the victims accountant. The victim testified that the had never been to her apartment before the night of the alleged incident. The came by to drop off some papers and then sexually assaulted the victim. The Prosecution offered evidence that the s fingerprints were found in the victims bedroom. The testified that a month prior, he and the victim had a consensual encounter at the apartment. The offered expert testimony that the fingerprints could have been there a month later. Was the trial judge right to

admit the s testimony about the alleged earlier encounter? Sexual activity with the same person again, may show consent. is offering evidence to show where the source of the damming evidence came from. Evidence to show semen or other physical evidence from someone other than the accused (a) He is not saying that they are not his, he is saying that they are old. would argue (c) evidence of the exclusion of which would violate the constitutional rights of the (due process). Under letter of law, this evidence would not come in, but some judges may allow it. Under strict reading of the FRE, the evidence would not come in, b/c the is not using it to show consent; he is using it to show where the evidence came from. On an essay, you would argue the rationale of the FRE for the , and give reasons why the evidence should come in. Problem 5.3 At the s new trial in Smith assume that the trial judge finds that reasonable jurors could find, based on the evidence presented by the , that the victim had made prior false allegations. Would the s proposed inquires also survive scrutiny under FRE 403, 404, and 608? FRE 608 goes to truthfulness. Therefore, evidence comes in b/c it shows that the accuser is untruthful. On cross-examination, is the only way that the victims character can come in if she testifies, and then he is stuck with the answer given by the victim. The 608 argument would be that false accusation comes into show that the victim is untruthful. FRE 404 character trial is inadmissible b/c if she lied in the past, it does not show that she had propensity to lie and lied in this instance. would argue that it shows the victims bias. Why she lied about the incident would come in as bias. Problem 5.4 The met the victim in a bar and offered her an invitation to his home. The stopped his car at a secluded stop and allegedly raped her. The sought to offer evidence that the victim had stated that she had been raped after engaging in consensual sexual intercourse. The s witness would testify that the victim had accused a friend of rape when they all knew that she had not been raped. If the Prosecutor objected to the s proposed evidence, should the trial judge nonetheless have admitted it? How might the court have distinguished this case from Smith? Assume the , instead of offering a separate witness to testify about this incident, proposed merely to cross-examine the victim about it. Would your ruling on admissibility change? Prosecution argues that it should not come in b/c it goes to her prior sexual history. If it was not a lie, then it was probably prior consensual sexual act. Evidence is probably not coming in b/c he said it was consensual if it is consensual sex, then it is not coming in. If it is consensual activity, it does not come in under 412. If its a false allegation, then it would come in. Judge determines if it is false allegation under conditional relevance of the Huddleston standard (preponderance of the evidence). Distinguished from Smith, b/c in Smith there was no prior sexual activity, there were only false allegations. Here, there was a prior sexual encounter (maybe consensual, maybe not), but in any case there was prior sexual activity that will be excluded under 412. Problem 5.5 Under the first bullet point the evidence would be inadmissible, because it would have going through the propensity box. Under the 2nd bullet point, the evidence would be admissible because it would prove he motive and bias by the prosecutions witnesses. Under the third bullet point it would have to satisfy the conditional relevance standard (Huddleston) for the judge to know if the jury by the preponderance of the evidence would believe that it led the victim to have PTSD. Problem 5.6 Paula Jones reputation only comes in if she puts it at issue. Argue FRE 403, likely it will not come in. President could argue FRE 412(b)(2) and claim that she put her reputation at issue. Why does the FRE sometimes treat and sexual history different? The rape shield laws are

designed to protect the victim and to encourage them to report rape, they were not designed to protect the . Competency of Witnesses In the past, felons could not testify; s could not testify; atheists could not testify Now, children are the primary focus: some children are not competent to testify; court must determine if they are competent to testify. Steps of court to determine competency of children: State v. Swan p. 332: ask child factual questions to determine if she knows the difference b/w a lie and the truth; she knew the difference, but did not have the competency to give correct factual answers; she really did not know the difference b/w being truthful and telling a lie. FRE 601: General Rule of Competency Every person is competent to be a witness except as otherwise provided in these rules. However, in CIVIL ACTIONS and proceedings, with respect to an element of a claim or defense as to which State law supplies the rule of decision, the competency of a witness shall be determined in accordance to state law (SMJ AND ERIE???-Civ Pro) Lack of Personal Knowledge Witness MAY NOT testify to a matter unless evidence is introduced sufficient to support a finding that witness has personal knowledge of the matter. Evidence to prove personal knowledge may but need not consist of the witnesss own testimony. This rule is subject to the provisions of FRE 703, relating to opinion testimony or expert witnesses (Huddleston standard) Oath or Affirmation Before testifying, every witness SHALL be required to declare that the witness will testify truthfully, by oath or affirmation administered in a form calculated to awaken the witness conscience and impress the witness mind with the duty to do so Religious Beliefs or Opinions Evidence of the beliefs or opinions of a witness on matters of religion is not admissible for the purpose of showing that by reason of their nature the witness credibility is impaired or enhanced (i.e. enhances credibility if witness is portrayed as a good Christian) 20 14; 75 51; 5 3 (Midterm)(B) (9/29/08 - 10/22/08) FRE on Hearsay Problem 7.1 Fearing death, the counsel gets an affidavit from their star witness stating that during the auto manufacturers own test the vehicles rolled over after exploding. Is the affidavit hearsay? The affidavit is a statement, made by the declarant out-of-court. This document is being offered to prove the truth of the statement and is therefore hearsay. Hearsay if it is was offered to prove the facts recorded in it; used to prove the truth of the matter asserted (own test of automaker showed hazard). If it was offered for some other reason than to show truth of matter asserted, it

FRE 602:

FRE 603:

FRE 610:

may be admissible. FRE 801(a) statement whether oral or written, if it is an assertion offered to prove truth of matter asserted, it is hearsay. Problem 7.2 To prove that Jeffrey was short on cash, the prosecutions witness offered a gesture [held up his right hand and rubbed his thumb together with his index and middle fingers] a gesture the witness recognized to mean money to explain why Jeffrey did not get a new bicycle. Is the gesture hearsay? YES, this gesture is a statement, made by the declarant out of court to prove the truth of the matter asserted. It does not have to be oral or written for the person to be asserting something, it can be a gesture. The gesture is non-verbal and he is asserting that he does not have any money. Problem 7.3 Witness is questioned as to what she said to detective. Is this hearsay? The fact that it was her statement does not disqualify it as hearsay. Witnesss testimony that she picked number 3 out of the line up is being offered to prove that she picked number 3, so that the detective could then testify that number three was the . This is NOT HEARSAY b/c it is covered by FRE 801(d)(1)(C). 801(d)(1)(c) allows this to come in one of identification of a person made after perceiving the person (non-hearsay). If it is offered for identification it is admissible admitted to prove was the one who robbed Alice (identification). While testifying, can testify to things the observed, but not to what they said. Problem 7.4 was charged with driving under the influence of alcohol and drugs. blood was tested and the print out shows that he had alcohol and PCP in his system. Is that printout of his results admissible in court? Yes it is hearsay Problem 7.5 If the s testimony in the James case (where pled self defense) after hearing of victims violent acts toward others) was objected to by the prosecutor, should the judge nonetheless have permitted it? The boasting can come in to show her state of mind. Not trying to prove what the person said to her. Only trying to use the statements to show that she was in fearstate of mind. Therefore, the statements are not being offered for the truth of the matter asserted. Problem 7.6 s testimony that she believed the horses belong to her friend was not hearsay. It was not offered to prove that he actually purchased the horses and asked her help in selling them. It was used for the purpose of showing her state of mind when she sold the horses. Out of court statement = yes, but they are not going to prove the truth of the matter asserted b/c it is going to show her state of mind. Problem 7.7 Kornberg [Volpes lawyer] testified that about a week before the end of Schwarzs trial, he had told Worth that my guy [Volpe] can take your guy [Schwarz] out of the bathroom. Kornbergs statement to Worth is not hearsay b/c it is offered to prove that Schwarz attorney (Worth) had knowledge of information that could have exonerated his client [Schwarz] but refused to call the witness [Volpe]. Worth did nothing about this knowledge. Schwarz alleged Worths failure to call Volpe as one basis for a finding of ineffective assistance of counsel. The statement IS NOT HEARSAY if offered by Schwarz to support his claim of ineffective assistance. The statement could have been offered to show knowledge by Volpe. If offered to show prejudice for a ineffective assistance claim, then it would be hearsay and would not come in. FRE 801: Definitions (a) Statement a statement is (1) an oral or written assertion or (2) nonverbal

conduct of a person, if it is intended by the person as an assertion (b) Declarant a declarant is a person who makes a statement (c) Hearsay Hearsay is a statement, other than one made by the declarant while testifying at the trial or hearing, (out of court statement) offering in evidence to prove the truth of the matter asserted. (d) Statements which are not Hearsay: A statement is not hearsay if (1) Prior Statement by witness: the declarant testifies at the trial or hearing and is subject to cross-examination concerning the statement, and the statement is (a) Inconsistent with the declarants testimony, and was given under oath subject to penalty of perjury at a trial, hearing, or other proceeding or in a deposition, or (b) Consistent with the defendants testimony and is offered to rebut or implied charge against the declarant of recent fabrication or improper influence or motive, or (c) Identification of a person made after perceiving the person; or (2) Admission by party-opponent: The statement is offered against a party and is (a) The partys own statement in either an individual or a representative capacity or (b) A statement of which the party has manifested an adoption or belief in its truth, or (c) A statement by a person authorized by the party to make a statement concerning the subject (statements by spokespersons) or (d) A statement by the partys agent or servant concerning a matter within the scope of the agency or employment, made during the existence of the relationship, or (e) A statement by a co-conspirator of a party during the course and in furtherance of the conspiracy. The contents of the statement shall be considered but are not alone sufficient to establish the declarants authority under (C), the agency or employment relationship and scope thereof under (D) or the existence of the conspiracy and the participation therein of the declarant and the party against whom the statement is offered under (E) Assertion - Something that you say or do with the expectation that the listener will believe The idea behind the hearsay rule is that having someone repeat another persons out of court statements is a poor way to have the jury find out the truth about the subject of the out of court words. Reasoning behind hearsay is b/c those who are NOT the declarant may have perception, memory, sincerity and narrative issues. (AKA testimonial infirmities) The rule leans towards admissibility. The burden is on the party claiming that the communicative intention existed; ambiguous and doubtful cases will be resolved against him and in favor of admissibility

Just b/c the declarant is the and testifies does not mean what they are going to say is NOT hearsay The exceptions to hearsay apply where there are reasons to think that the out of court statement was particularly likely to be accurate or truthful so that the impossibility of probing the original speakers statements is not so harmful Truth of the matter asserted = realitybeliefstatement; but not to prove that the statement was true but that it was made Ex. To prove that the apt was cold 1. It is very cold in here HEARSAY 2. This is a great place for polar bears HEARSAY 3. I need to put on a sweater NOT hearsay; not trying to prove the truth of the matter that the apt is cold (verbal conduct which is assertive but offered as a basis for inferring something other than the matter asserted is ALLOWED) Hearsay can NOT be taken from animals or mechanical devices A declarants words cannot be hearsay if it is in a form of a question What is considered to not be a statement An order is not a statement A question is not a statement Talking in your sleep- not a statement b/c no intent to communicate Involuntary behavior not a statement b/c no intent to communicate FRE 801(d)(1) and 801(d)(2) are NOT exceptions to the hearsay rule; they consider those statements to not fall under the hearsay rule at all; Personal knowledge is not required for this rule FRE 801(d)(1)(B) has to satisfy that (i) the declarants testimony has been attacked as fabrication for a motive to lie & (ii) the witness made the prior statement BEFORE the time of the alleged fabrication or before the motive to lie. IF the statement were made AFTER the motive to lie, then it would NOT be eligible for NONHEARSAY treatment FRE 801(d)(1)(C) has the rationale that when identification is made before the trial outside the presence of influence right after the crime has been committee then it is more likely to be accurate and thus should NOT be excluded as hearsay for substantive use FRE 801(d)(2) should pay attention to the words and not it being admissions; that the partys own words from himself; agent; spokespersons and co-conspirator can be used against them rationale behind this is that a person will not lie to hurt

themselves; cross examination is possible against those declarants who testify in court and the last reason is b/c this system is adverse (i.e. this is war) FRE 801(d)(2)(A) Though the Constitution protects against coerced confessions, the hearsay rule poses no obstacle to confessions made freely. This FRE declares that a partys own words are not-hearsay when offered against her at trial. FRE 801(d)(2)(B) - Adoptive admission conditions (1) Party hears and understands the statement (2) At liberty to respond (3) Circumstances naturally call for a response (4) Failure to respond To establish co-conspirator under FRE 801(d)(2)(E) they have to meet the following requirements: (1) conspiracy existed at the time the out of court statement was made (2) that the conspiracy included both the declarant and the party against whom the statement is offered (3) that the declarant spoke during the course of and in furtherance of the conspiracy; in order to bring this in FRE 104(b) will be applies (Huddleston Standard) also that the hearsay evidence ITSELF could be used as the preponderance of evidence to be brought into trial FRE 801(d)(2)(E) is the use of 104(a) in order for the judge to decide if the preliminary conditions have been met (preponderance of evidence) before using the statement; however the court can not only use the statement in order to pass the preliminary conditions FRE 802: Hearsay Rule Hearsay is not admissible except as provided by these rules or by other rules prescribed by the Supreme Court pursuant to statutory authority or by Act of Congress. The hearsay FRE is about the reliability of the evidence that the jury hears. For example, when a witness testifies, Then I saw John pull the trigger, we worry about a failure of one or more four testimonial capacities. The information the jury hears about the shooting is accurate only if all four of the wittinesss testimonial capacities are sound. 1) Perception: The witness saw Tom pull the trigger, but mistook him for John 2) Memory: The witness saw and recognized Tom, but now thinks it was John 3) Narration: The witness means to say Tom, but says John 4) Sincerity: The witness means to deceive Hearsay is an out-of-court statement offered by a litigant to prove what the statement asserts. To decide whether any particular out-of-court statement is hearsay, we must ask: 1) Is the litigant offering the statement to prove (the truth of) what it says? 2) Is it an assertion? Unless the answer to both questions is yes, the statement is not hearsay. When is something NOT hearsay If it fails any part of FRE 801

No statement b/c there is no assertion Exceptions are 803 & 804 (i.e. effect on the listener/reader, words of substantive legal significance, statements offered to impeach a witness) No offering to prove the truth of the matter asserted What makes testimony hearsay? A statement or assertion Made by declarant out of court That is offered to prove the truth of the matter asserted Problems with hearsay: No opportunity to cross examine declarant No opportunity to test memory, perception, etc. Lack of ability to observe witness No oath Lack of cross-examination FRE 805: Hearsay within Hearsay Hearsay included within hearsay is not excluded under the hearsay rule if each part of the combined statements conforms with an exception to the hearsay rule provided in these rules Attacking and supporting credibility of declarant When a hearsay statement, or a statement defined in FRE 801(d)(2), (C), (D), or (E), has been admitted in evidence, the credibility of the declarant may be attacked, and if attacked may be supported, by any evidence which would be admissible for those purposes if declarant had testified as a witness. Evidence of a statement or conduct by the declarant at anytime, inconsistent with the declarants hearsay statement, is NOT subject to any requirement that the declarant may have been afforded an opportunity to deny or explain. If the party against whom a hearsay statement has been admitted calls the declarant as a witness, the party is entitled to examine the declarant on the statement as if under cross-examination Declarant can be attacked as if a witness under FRE 608, however can be supported by extrinsic evidence. If the declarant makes a statement inconsistent with their hearsay statement they dont have the chance to explain themselves. Also under this rule, if the party of whom the statement is against calls the declarant a witness, they can treat them as if they were being cross examined on the stand Is it an assertion? Conduct may or may not be an assertion The declarants intent controls the inquiry (communicative intent) (i.e. CAN: nothing is an assertion unless intended to be one) Could the conduct be a lie? (A man does not lie to himself) Defining Assertions: Problems

FRE 806:

#1 Hearsay b/c the minister is asserting that the beef is safe. He is telling the public its safe. #4 Hearsay b/c evidence is offered to prove the officer is making an assertion thru his conduct (shooting at the guy who shot at him). C/O not hearsay--the police officer is not asserting anything (NOT is the person who shot me)! He is trying to protect other people by shooting at the fleeing felons. (Judge FREs on preponderance of the evidence standard.)its an observation; relevant. #5 Declarant = the anonymous caller. I want to . .. out of ct stmt. Want to offer it to show it was a gambling den. Argueit is not an assertion. The declarant is trying to place a bet, he is not trying to est. anything b/c the declarant already know that its a gambling place (not asserting that it is a gambling place) C/O it is an assertion that the place is one that receives bets. Argue declarant is asserting this is a gambling place. Ct will likely find it is not an assertion. # 6 Hearsay b/c this is an assertion that Nelson was the attacker. Victim intended to communicate-Intending to assert to the police that Nelson stabbed him. In this case, the police were present to overhear it. C/O argue the victim is traumatized so unreliable, 403 prejudicial. Ct would probably not let the stmt in. (may come in since its an ID) #7 Declarant is Parran; he is asserting I didnt tell them anything about you. Parran is asserting that the other guy is involved=hearsay. The implication is that he was involved. C/O Parran is not asserting b/c he already knew he was involved unless he knew the police officer could hear him; then he is making the stmt to the police officer to get the other guy in trouble. # 8 She wrote it in her diary; no audience. Unless she knew the diary would be found, and she wrote it on purpose (i.e. the diary is laying out, open to that page) v. key is hidden, under mattress. She is not trying to assert anything. Theorywe dont lie to ourselves. Argue: not an assertion, not hearsay. Exceptions to the Hearsay FRE:
(1) FRE 801(d)(1): Prior Statements by Witnesses (A) (B) (C) (2) FRE 801(d)(2): Prior Inconsistent Statements Prior Consistent Statements Statements of Identification Non-Hearsay

Admissions by Party Opponents (A) (B) (C) (D) (E) The Partys Own Statement Adoptive Statements Statements by Spokespersons Statements by Agents Co-conspirators Statements

(3) FRE 803:

Exceptions in which the Availability of the Declarant is Immaterial (1) (2) (3) Present Sense Impressions Excited Utterances Then-Existing Mental, Emotional, or Physical Conditions

(4) (5) (6) (7) (8) (10) (4) FRE 804:

Statements for Medical Diagnosis Recorded Recollections Business Records Absence Business Records Public Records and Reports Absence Public Records and Reports

Hearsay Exceptions

Exceptions Applicable Only When the Declarant is Unavailable (b)(1) Former Testimony (b)(2) Dying Declarations (b)(3) Statements Against Interest (b)(4) Family/Personal Information (b)(6) Forfeiture by Wrongdoing

(5) FRE 807:

Residual Exception

Statements of Party-Opponents (see above) The Partys Own Words Focus on 801(d)(2)(A) Problem 7.11 L sues an airline after a rough landing, claiming that resulting injuries prevented her from working to full capacity. offers records signed by L showing that she billed clients an average of 104 hours per week following the accident. Are the records admissible over Ls hearsay objection? Yes, under 801(d)(2), the records are the partys own statements. Problem 7.12 O.Js statements Well take my blood test and we will see was said by the defendant, however under 801(d)(2), it must be offered by the opposing party. Since Shapiro (s counsel was trying to get it in and not the prosecution) it was not admissible. Shapiro wants it in to show that OJ was offering his services. Sustained b/c it is OJs statement being offered by his counsel and not the opposing counsel. Adoptive Statements (see above) Problem 7.13 Undercover officer arrested two people for drug possession, with the intent to distribute. Prosecution sought to admit the statement I only have one, but you can get another from my buddy to convict Beckham who was sitting on a nearby bench, and at that moment got up and pulled out a Ziploc bag with drugs. This is hearsay, but it does not get in under exception 801(d)(2)(e) (b/c the court said there was insufficient evidence), which states that a co-conspirator of a party during the course and in furtherance of the conspiracy. It can get in under 801(d)(2)(b). In order for this adoptive admission to apply, the party has to hear the statement, be at liberty to respond, circumstances must call for a response and the party failed to respond. Beckham by not responding adopted the statement. By not saying anything you are adopting the statement as your own. Problem 7.14 Daughter visits her father in jail where he is serving time for the murder of her childhood playmate. Daughter discusses telling the truth to which her father responds by pointing to a camera, meaning that they were monitoring the conversation. He does not deny the accusations. Prosecutor seeks to admit this evidence as an adoptive admission. Could this testimony be admitted? Here it could be argued that the father was not at liberty to respond since he knew and indicated

that they were being monitored. He had also been Mirandized, giving him the right to remain silent. Not adoptive under FRE 801(d)(2)(B). Reversed on 5th amendment. Statements of Agents (see above) Problem 7.15 fell on an icy walkway. Later on a friend telephoned the management company to warn them of the icy conditions. A man with a shovel and a bucket appeared. Assume the man told the friend that the guys on the day shift were supposed to shovel and salt, but they left early. Should the court admit this evidence against the condominium? Yes, the shoveler was acting within the scope of his employment and therefore these statements should be admitted under 801(d)(2)(D) b/c the statement is not hearsay. The fact that the identity of the shoveler was not disclosed does not render his statement invalid. 801(d)(2)(E) and 104(a) (see above) (1) Co-conspirator must be one of the declarants (2) During the course of the conspiracy (3) In furtherance of the conspiracy The Existence of a Conspiracy The co-conspirator exception almost never applies to a confession made knowingly to the police and implicating ones associates. The FRE requires the statement to have been made during the course and in the furtherance of the conspiracy. Such a confession may well terminate the conspiracy. It never can qualify as furthering the conspiracy. Distinguishing 104(a) Questions from 104(b) Questions - How do we know which preliminary questions the judge should decide by a preponderance of the evidence under 104(a) and which should submit to the jury under 104(b)? The clearest answer is that FRE 104(a) suggests that all preliminary questions are to be resolved by the court subject to the provisions of subdivision (b). And FRE 104(b) address only those preliminary questions upon which the relevancy of evidence depends. Under 104(a) the judge can look at inadmissible and admissible evidence; under 104(b) the jury can only look at the admissible evidence. That is, FRE 104(b) governs matters of conditional relevance, and FRE 104(a) governs everything else (determinations of admissibility/competency of evidence). Examples of 104(b) is FRE 801(d)(2)(B), FRE 404(B) Problem 7.16 Translation: This is hearsay within hearsay, b/c it was relayed from a translation. What exception does this come in under? There is key evidence to support that a conspiracy occurred. The court has to decide under 104(a) based on preponderance of the evidence that the translation was done sufficiently. Then they would have to go through the factors of the conspiracy b/w the men and then be able to bring in the statements, which would not fall under hearsay within hearsay by FRE 801(d)(2)(e) Past Statements of Witnesses and Past Testimony:
FRE: Topic: Conditions Regarding Declarants Availability or memory Conditions regarding past statement

613

Past inconsistent statements offered to impeach Past inconsistent statement offered substantively

Declarant must have testified declarant must testify at trial or hearing and be subject to cross examination concerning the statement Same

801(d)(1)(A)

Questioning Lawyer must have good-faith belief that the witness made the past statement Past statement is inconsistent and was Given under oath and; At a proceeding or Deposition Past statement is consistent, is offered to rebut charge of recent fabrication or improper motive and meets Tome FRE Past statement identifies a person and was made after declarant perceived the person Past statement was 1. testimony, 2. at a proceeding or deposition, and 3. subject to examination by party against whom now offered (or by a civil predecessor in interest) who then had similar motive None (note that memory may be refreshed with many things; if a writing is used, FRE 612 imposes conditions Record: was made or adopted when witnesses memory was fresh; reflected witnesses knowledge correctly.

801(d)(1)(B)

Past consistent statement

801(d)(1)(C)

Statement of identification Past testimony

Same

804(b)(1)

Declarant must be unavailable as defined by FRE 804(a)

612

Refreshing witnesses memory

Witness must be on the stand; memory must be exhausted

803(5)

Past recollection recorded

Witness must be on stand; must have insufficient recollection

FRE 613: Inconsistent Statements Offered to Impeach FRE 613: Prior statements of Witnesses (a) Examining witness concerning prior statement in examining a witness concerning a prior statement made by the witness, whether written or not, the statement need not be shown nor its contents disclosed to the witness at that

time but on request the same shall be shown or disclosed to opposing counsel (b) Extrinsic Evidence of prior inconsistent statement of witness extrinsic evidence of a prior inconsistent statement by a witness is not admissible unless the witness is afforded an opportunity to explain or deny the same and the opposite party is afforded the opportunity to interrogate the witness thereon, or the interest of justice otherwise require. This provision does not apply to admissions of a party opponent as defined in rule 801(d)(2) This WILL NOT come in as substantive evidence, but it will come in as substantive evidence under FRE 801(d)(1)(a) or (b). If you cannot get it in under FRE 801, you may be able to get it in under this rule. When prior statements are offered to impeach, they are not offered for the truth of what they assert, but merely to show that the witnesses says different things at different times and therefore should not be believed. This rule is NOT subject to FRE 405 limits, so extrinsic evidence of the prior inconsistent statements is ALLOWED, but prior inconsistent statements offered under FRE 613 are only admissible to show that witness is not to be believed, not as substantive evidence Problem 7.17: Retraction: this would not come in under FRE 801(d)(1)(A), but it could come in under FRE 613 b/c all the prosecution did was impeach her, and the substantive evidence was not counted. So based on the JOA, the judge would grant the motion. Problem 7.18: Why Such Excitement? After F and G were arrested on suspicion of involvement in a drug-running operation, a Customs agent overheard G say to F, Why so much excitement? If they caught us w/ the goods, they caught us w/ the goods. F remained silent. At trial, may the govt offer evidence of Gs statement, together w/ Fs silence. As substantive evidence? No, According to 801(d)(2)(B), Fs silence may be considered an adoptive admission. However, it must meet the following elements. (1) the statement was heard and understood; (2) the party was at liberty to respond; (3) the circumstances naturally called for a response; (4) the party failed to respond. This really depends on if he had been read Miranda. If he had not, it would likely come in. If the statements are made in a private setting even if he is in custody, it still comes in. Another possibility is co-conspirator, but that will not work, b/c the statement was not made in furtherance of selling drugs. To impeach F if he testifies and denies all knowledge of the drug operation? No, he made no assertion Hearsay: Miranda and Silence: Silence sometimes constitutes an adoption under FRE 801(d)(2)(B). If someone says, I hear you robbed that bank, we would expect an innocent person to deny the allegation. There are four preconditions to using silence as evidence of an adoptive admission: i. The statement was hear and understood by the party against whom it is offered; ii. The party was at liberty to respond iii. The circumstances naturally called for a response; and iv. The party failed to respond Silence may also be used to impeach if the witnesss previous silence is inconsistent with his testimony on the stand. Again, one must ask if it would have been natural, under the circumstances,

to assert the fact at issue. Although the Supreme Court has not ruled whether post-Miranda silence may serve as evidence of an adoptive admission, the answer is almost certainly NO. The Supreme Ct has held that in the absence of Miranda warnings, a s silence may be used to impeach the s testimony, whether or not the was in custody at the time of the silence in question. Defendant not in Custody May silence by an adoption Under FRE 801(d)(2)(B) May Silence be Used to Impeach? ?? Circuits are Split YES: Jenkins v. Anderson Defendant in Custody/PreDefendant in Custody/ Post

Miranda

Miranda
NO

?? (YES: US v. Frazier) (NO: US v. Flecha: U.S. v. Velarde-Gomez YES: Fletcher v. Weir

NO: Doyle v. Ohio

Miranda Warnings, Adoptive Admissions and Impeachment If is NOT in custody, the courts are split as to whether silence is an adoptive admission. If is NOT in custody, silence may be used to impeach the . If the is in custody and they HAVE NOT been read their Miranda Rights, Silence may NOT be an adoptive admission. If the is in custody and they HAVE NOT been read their Miranda Rights, Silence may be used to impeach. If you HAVE been read your Miranda Rights, Silence may NOT be an adoptive admission If you HAVE been read your Miranda Rights, Silence may NOT be used for impeachment. FRE 801(d)(1)(A): Inconsistent Statements Offered Substantively Problem 7.19 FT states that her boyfriend hit her in the eye with his open hand at the grand jury testimony. Now at the trial she recants and says it was an accident. The prosecution offers the previous testimony over the s objections. Is it admissible without any other evidence The court would state that it can be admissible to show that it was prior inconsistent testimonies under 801(d)(1)(A). So it would not apply to the hearsay at all. However the court has to look at if 104(a) has to be applied Problem 7.20 The same above facts except FT does not state it was an accident but she lost her memory b/c of the injury. Can the prior testimony be admissible. Under this case, it would be hearsay b/c it was an out f court statement made by the declarant to prove the truth of the matter asserted. The court would states that they would need it to refresh the witnesses memory under FRE 613 it does not need to be entered into evidence. But can be used to refresh the memory of the witness. FRE 801(d)(1)(B): Past Consistent Statements FRE 801(d)(1)(C): Statements of Identification FRE 804: Hearsay Exceptions; Declarant Unavailable (a) Definition of unavailability. Unavailability as a witness includes situations in which the declarant

(1) Is exempted by ruling of the court on the ground of privilege from testifying concerning the subject matter of the declarants statement; or (2) Persists in refusing to testify concerning the subject matter of the declarants statement despite an order of the court to do so; or (3) Testifies to a lack of memory of the subject matter of the declarants statement; or (4) Is unable to be present or to testify at the hearing b/c of death or then existing physical or mental illness or infirmity; or (5) Is absent from the hearing and the propend of a statement has been unable to procure the declarants attendance (or incase of a hearsay exception under subdivision (b)(2),(3), or (4) the declarants attendance or testimony) by process or other reasonable means A declarant is not available as a witness if exemption, refusal, claim or lack of memory, inability, or absence is due to the procurement or wrongdoing of the proponent of the statement for the purpose of preventing the witness from attending or testifying (b) Hearsay Exceptions The following are not excluded by the hearsay rule if the declarant is unavailable as a witness: (1) Former Testimony. Testimony given as a witness a another hearing of the same or different proceeding, or in a deposition taken in compliance with law in the course of the same or another proceeding, if the party against whom the testimony is now offered, or, in a civil action or proceeding, a predecessor in interest, has an opportunity and similar motive to develop the testimony by direct, cross, or redirect examination (2) Statement under belief of impending death. In a prosecution for homicide or in a civil action or proceeding, a statement made by a declarant while believing that the declarants death was imminent, concerning the cause or circumstances of what the declarant believed to be impending death (3) Statement against interest. A statement which was at the time of making do far contrary to the declarants pecuniary or propriety interest, or so far tended to subject the declarant to civil or criminal liability, or to render invalid a claim by the declarant against another, that a reasonable person in the declarant position would not have made the statement unless believing it was true. A statement tending to expose the declarant to criminal liability and offered to exculpate the accused is not admissible unless corroborating circumstances clearly indicate the trustworthiness of the statement (4) Statement of personal or family history. (A) A statement concerning the declarants own birth, adoption, marriage, divorce, legitimacy, relationship by blood, adoption, or marriage, ancestry, or other similar fact of person or family history, even though declarant had no means of acquiring personal knowledge of the matter stated; or (B) A statement concerning the foregoing matters, and death also, of another person, if the declarant was related to the other by blood, adoption, or marriage or was so intimately associated with the others family as to be likely to have accurate information concerning the matter declared (5) Transferred to FRE 807 (6) Forfeiture by wrongdoing. A statement offered against a party that has

engaged or acquiesced in wrongdoing that was intended to, and did, procure the unavailability of the declarant as a witness Under FRE 804(b)(1) similar motive in direct, cross and redirect has to be almost the same as the reason for their testimony and why it is being offered in trial. Currying Favor Statements: When a begins to implicate others in an effort to receive an easier jail term or plea-bargain; then those statements cannot be admitted b/c there is no guarantee as to their trustworthiness Under FRE 804(b)(2) the judge has to decide based on preponderance of the evidence that the declarant was impending death when making their statement, in which it follows FRE 104(a); NOT 104(b) where the question would be given to the jury to decide if it was conditional that death was impending in order for the statement to be admissible Under FRE 804(b)(6) uses a 104(a) standard to determine if the caused for the declarant to be unavailable for trial FRE 804(a) and 804(b)(1): Past Testimony (see above) Problem 7.22 Spousal Abuse IV: Wife testified that her husband hit her at the grand jury proceeding, but at the trial, she would not testify that he had hit her. She was not subject to cross examination at the grand jury. Since she is refusing to testify despite the judges threat to hold her in contempt, she is unavailable under FRE 804(a)(2). She would not take the stand, the former testimony was inadmissible under FRE 804. FRE 807: Residual Exceptions: her testimony would probably come in under FRE 807. Person trying to admit the evidence must give notice that they plan on admitting this evidence. Problem 7.23 Crewing was charged with a DUI in an accident involving Morgan. Morgan sued Crewing for injuries from the accident. At the civil trial Morgan testified about the accident and was cross-examined by Crewings lawyer. Later, Crewings DUI came to trial. By that time, Morgan had suffered a serious head injury and couldnt attend the trial. The prosecutor wanted to read into the record Morgans testimony at the earlier trial, and Crewing objected. The motive to cross-examine in the civil trial is not the same as the motive in the criminal trial. The penalties and burden of proof differ from civil to criminal. She is unaviable based on FRE 804(a)(4) and the former testimony would be admissible under FRE 804(b)(1). If the person is unavailable b/c of a mental infirmity, and the previous testimony was subject to cross-examination it would probably come in. The defense attorney could argue that the attorney had a different motive in examining the witness than in the criminal trial. The court might look at the rest of the record and see the extent of her crossexamination. FRE 804(b)(3): Statements Against Interest (see above) Problem 7.24 Ask Magnolia: The evidence should only be admissible under FRE 804(b)(3) as a statement against the declarant only if there was evidence to corroborate that it was Magnolias idea. It would not be against her, b/c there was no evidence to corroborate it. If would come in against him as an admission/statement against his interest. under 804(b)(3). Declarant Unavailable FRE 804 (b) Hearsay exceptions

Exception for former testimony Dying declarations (homicide or civil case) o Declarant must believe death imminent o Must concern circumstances of impending death o Declarant DOES NOT have to die. Statement Against Interest Statement of Personal or Family History Forfeiture by Wrongdoing

Problem 7.24 A man and woman team robbed an armored truck. After Magnolia was arrested, the police acted on a tip and went to the home of Robert Barton. Bartons mother answered the door and asked her son Bobby, did you rob that truck? Dont lie to me. Barton replied Ask Magnolia. At the joint trial of both , the prosecutor seeks to offer this conversation into evidence. Assume that Bartons invocation of the 5th Amendment makes him unavailable. Is this admissible? No, under 804(b)(6) this statement has led to the s making himself unavailable and therefore should not be admitted. Problem 7.25 Alices Restaurant: In this particular exception, you first have to find that the witness is unavailable. Acker in this problem is unavailable b/c he has a Fifth Amendment privilege. He is allowed to say the things that incriminate him, but he is not able to say things that incriminate Alice based on FRE 804(b)(3) Problem 7.26 Accomplices: Barone and Limoli are charged with robbery. During the robbery, McPhee, a security guard was shot in the neck and foot. Witness Jordan testified that Barone told him that he and Limoli were responsible for the robbery, and that Barone had shot the security guard in the neck. (This statement was allowed in under 801(d)(2) a statement of a party opponent). DiNunzio, sister of Limoli testified that her brother came to her house the morning of the robbery and told her that they had committed the robbery, and that he (Limoli) had shot McPhee in the foot, while Barone had shot McPhee in the neck. Should the trial court have admitted DiNunzios testimony over a hearsay objection? Yes, this statement should be admitted under 804(b)(3) since this is a statement against interest by Limoli who is now unavailable to testify. The 1st circuit allowed the entire statement in. Although, there is a VERY strong argument that portions of the statement should have been excluded. Problem 7.27 Bucky wasnt involved: The statement would be admissible against Tilley, but it should not be admissible to protect Bucky b/c it is incriminating Buzzy. FRE 804(b)(2): Dying Declarations (see above) Problem 7.28 Mullen was on his death bed, so badly that the doctor regarded him as hopeless. The mother of the was present during his final hours, where Mullen stated I know your son, Clyde Mattox and he did not shoot me. The sought to admit this statement into evidence. This question was objected as incompetent and the objection sustained. Was this correct? No, this should have been overruled, since Mullen did not have a time line as to how long he was going to live. Defense can argue people are more likely to tell the truth when they know theyre about to die, so statements made impending imminent death are reliable. As it would satisfy the FRE 804(b)(2) FRE 804(b)(2): Statement under belief impending death (see above)

1) Believe death is imminent 2) Must concern circumstances declarant believed to be impending death FRE 804(b)(6): Forfeiture By Wrongdoing (see above) In order to satisfy this rule, 104(a) has to be satisfied, in such that by the preponderance of the evidence the in some way caused for the witness to be unavailable Hearsay Exceptions Under FRE 803: Availability of Declarant Immaterial 1) Present Sense Impressions 2) Excited Utterances FRE 803: Hearsay Exceptions; Availability of a Declarant Immaterial The following are NOT excluded by the hearsay FRE, even though the declarant is available as a witness: (1) Present Sense of Impression: A statement describing or explaining an event or condition made while the declarant was perceiving the event or condition, or immediately thereafter. (2) Excited Utterance: A statement relating to a startling event or condition made while the declarant was under the stress of excitement caused by the event or condition. (3) Then existing mental, emotional, or physical condition: A statement of the declarants then existing state of mind, emotion, sensation or physical condition (such as intent, plan, motive, design, mental feeling, pain and bodily health), but not including a statement of memory or belief to prove the fact remembered or believed unless it relates to the execution, revocation, identification or terms of declarants will (4) Statements for purposes of medical diagnosis or treatment: Statements made for purposes of medical diagnosis or treatment and describing medical history, or past or present symptoms, pain or sensations, or the inception or general character of the cause or external source there of insofar as reasonably pertinent to diagnosis or treatment (5) Recorded Recollection: A memorandum or record concerning a matter about which a witness once had knowledge but now has insufficient recollection to enable him to testify fully and accurately, shown to have been made or adopted by the witness when the matter was fresh in the witnesss memory and to reflect that knowledge correctly. If admitted, the memorandum or record may be read into evidence but may not itself be received as an exhibit unless offered by an adverse party. (6) Records of regularly conducted activity (business records) A memorandum, report, record, or data compilation, in any form, of acts, events, conditions, opinions, or diagnosis made at or near the time by, or from information transmitted by, a person with knowledge, if kept in the course of regularly conducted business activity, and if it was the regular practice of that business activity to take a memorandum, report, record or data compilations all as shown by the testimony of the custodian or other qualified witness, or by certification that complies with FRE 902(11) and 902(12) or a statute permitting certification, unless the source of information

or the method of circumstance of preparation indicate lack of trustworthiness. The term business as used in this paragraph includes business, institution, association, profession, occupation, and calling of every kind whether or not conducted for profit. (7) Absences of entry in records kept in accordance with business records: Evidence that a matter is not included in the memoranda, reports, records, or data compilations, in any form, kept in accordance with FRE 803(6), to prove the nonoccurrence or nonexistence of the matter, if the matter was of a kind of which a memorandum, report, record or data compilation was regularly made and preserved unless the sources of information or other circumstances indicate lack of trustworthiness (8) Public records and reports: Records, reports, statements, or data compilations in any form of public offices or agencies setting forth (A) The activities of the office or agency, or (B) Matters observed pursuant to duty imposed by law as to which matters there was a duty to report, excluding, however, in a criminal cases matters observed by police officers and other law enforcement personnel, or (C) In civil actions and findings resulting from an investigation made pursuant to the authority granted by law, unless the source of the information or other circumstances indicate lack of trustworthiness. (10) Absence of Public record or entry: To prove the absence of a record, report, statement, or data compilation, in any form, or the nonoccurrence or nonexistence of matter of which a record, report, statements or data compilation, in any form, was regularly made and preserved by a public office or agency, evidence in the form of a certification in accordance with FRE 902, or testimony, that diligent search failed to disclose the record, report, statement, or data compilation, or entry. Under FRE 803(1) time is critical with a present sense impression Under FRE 801(2) One court has held that up to 8 hours is still an excited utterance. A Judge will determine under 104(a), whether it is an excited utterance Under FRE 803(3) that statements rendered by the declarant should be admissible only to prove his future conduct not the future conduct of another person (Hillmon); (Hillmon stated that a declaration of intention to prove the past have inherent risks but declaration of intention of the future have no risk as to hearsay) Under FRE 803(5), there is predicate that has to be laid out before the evidence can be used to refresh the recollection of the declarant (1) Personal knowledge, (2) At or near the time of the incident fresh (3) Insufficient recollection at the time of testifying; (4) Witness has to vouch for the accuracy of the document; (5) The person who made the past statement must be presently on the witness stand; (6) Refreshing recollection must be attempted first and be shown to have failed. The writing must be authenticated.

FRE 803(1) & (2) Present Sense Impressions and Excited Utterances Problem 7.29 Dog Mauling. In a lunchtime phone call to her domestic partner, Whipple exclaims that dog just bit me I told Noel (dog owner) You need to control your dog! Are these excited utterances? Most likely yes. Time is not a factor and Whipple could still likely be under the stress of being bit, especially since in her phone call she said that dog JUST bit me. Statements made after her domestic partner got home, most likely does not meet excited utterances. Problem 7.30 Dog Mauling II: 911 call in which a neighbor states that she hears the dogs owner screaming in the hall way. She thinks the dogs owner is being attacked and that she doesnt dare go out there b/c the dogs are ferocious and huge. The seeks to admit this tape as evidence that she was trying to control the dog. What is the problem with present sense impression? There is no indication or facts to support that she can see outside into the hallway, and can actually see what is going on. In this case the tape was not allowed into evidence. Most judges would find that there is no preponderance of the evidence it would probably not come in. You could argue present sense impression in regard to the witness. This would be more reliable b/c she knows the dogs and the neighbors voice. Evidence is more likely to come in under this argument of present sense impression. What about excited utterance? Yes, she was under the excitement of the event. The timeframe is clear. Have to prove to the court under 104(a) by a preponderance of the evidence whether this is an excited utterance. More than likely, the woman will have to be in court to tell how she was scared, etc. Problem 7.31 Husband and wife got into an argument. The police were called and upon their arrival, the wife related that her husband had knocked her down and pushed her head into a wall. At trial, Lori admitted that the two had argued but denied that her husband had ever pushed her. During the states case, they offer Officer Martin who testifies that soon after the argument Lori told him her husband had pushed her. If defense counsel objects, is the trial judge correct in admitting the evidence? Problem with a present sense impression: It was not immediately after the eventneeds to be right at the moment, not following a fight and waiting for the police to get there. Excited Utterance: More likely admitted under excited utterance, since Lori could have still been under stress of excitement of the fight indicated by her crying and anxiousness. Problem 7.32 Bartender heard a shot, then a scream. Someone then ran in and told her to telephone the police. After doing so she went outside and asked Who had shot the gun. No one answered her. Bonnie Eaton then yelled out Joe Puleio. The objected to both statements concerning that Bonnie Eaton yelled out and then naming the . Was the trial judge right to admit bartenders response? Defense could argue that it should not come in as a present sense impression since the bartender had no way of knowing whether the person that yelled out was even there. Under excited utterance, prosecutor would argue that little time had passed, and therefore one could assume that the witness was under the emotional stress of seeing this. Probably admitted as an excited utterance. FRE 803(3) Statements of Then-Existing Condition Problem 7.33 Larry disappeared after making statements that he was going to meet Angelo in the parking lot to pick up a pound of marijuana. Court admits statements from Larrys girlfriend and friend stating that he [intended] was going to the parking lot to pick up the marijuana. These statements can be admitted to show that a person (in this case) Larry acted in conformity with his

intent, but not that Angelo did anything to Larry. (Hillmon doctrine can apply here) Shepard v. United States Used for dying declaration FRE 803(4) Statements for Medical Diagnosis Problem 7.34 Elder Abuse: The lawyers testimony that Browning said he had fallen and hit his headNO, when a statement is not used for diagnosis and is not a present sense expression since he is going back, this can not be allowed in. More than likely an excited utterance but time is remote. The doctors testimony that Browning said he had fallen and hit his headYES, this is exactly what the FRE is designed to admit. The doctors testimony that Browning said Maples had pushed himMAYBE, if his attorney can argue that knowing who did it (his caretaker) is vital to his health and should thereby be removed it could be allowed in under 803(4) to prove diagnosis. Problem 7.35 Child Abuse I: When the doctor questioned the child about what happened to his arm, the child statedI was playing with one of my fathers records, got it dirty and he twisted my arm. While the trial court allowed it, the Court of Appeals reversed since I was playing with one of my fathers records and got it dirty was not pertinent to the diagnosis and a broken arm could have been proved by other evidence. Problem 7.36 Child Abuse II: Psychologist was cross examined in Tome as to whether she had discussed truth telling with the child witness. The psychologist said no. How does this affect the testimony under FRE 803(4)? It should not affect the testimony, b/c the statements made by the child, if admissible under FRE 803(4) would have to be for the purposes of a medical diagnosis or treatment. The FRE does not say anything about truth telling. Problem 7.37 Food Poisoning: Monicas husband was lying on the bed and said I think I ate some bad meat. youd better get me a doctor. She saw an empty take out container from the down town deli. He later died of arsenic poisoning. Monica sued the down town deli for negligent supervision of a disturbed employee. At trial she offers the following evidence. Her own testimony about her husbands statements and gesture offered to prove that he had eaten food from the Down town deli. - - admissible. A nurses testimony that when Monica called the doctors office, she said, my husband told me he ate some bad meat from the downtown deli admissible. Monicas own testimony that when the doctor arrived at their home, the doctor said, this has all the signs of arsenic poisoning. - - admissible. (do predicate questions for this) FRE 803(5) & 612 Refreshing Memory and Recorded Recollections Refreshing Memory(Recollection): (1) Insufficient memory (2) Give them something and ask, did that refresh your memory (3) If they do not remember, you cannot get it in. (this is unlike recorded recollections and those can be read into evidence.) Distinguishing Refreshing Recollection from Past Recollection recorded: (1) FRE 612 provide PR protections to the opponent of a party that attempts to refresh a Ws recollection through a writing

(2) Distinguish refreshing recollection from past recollection (3) PRRR used when Ws testimony is not revived (4) Then Ws needs to testify that Problem 7.38: License Plate: Menadier and Sullivan witnessed a car accident and wrote the license number down on a candy wrapper. At trial, witness says that she remembers writing down the plate number but she does not remember what the number is. The prosecution tries to offer the exhibit into evidence. The evidence should be able to be read into evidence, but should not be admitted as an exhibit and the writing must be authenticated. In this case, it is very possible that even the number written on the wrapper is not the correct number, so it may not be proper evidence. The judge should allow the evidence to be used to refresh Menadiers memory just as it was used for evidence. It can only be offered as an exhibit by an adverse party. Business Records Focus on 803(6) & (7) Problem 7.39 Lawnmowers: Donohue returned a lawnmower that was defective. Should her return receipt which the store clerk marked with the reason (defective power cord) be allowed in under the business records exemption? Yes, these records were taken in the ordinary course of business and should be admitted. If the receipt contained a statement to the effect the customer says the power cord is faulty, you should try to admit, however if opposing counsel objects under FRE 805, you should redact the receipt to allow the other portions in. This cant come in as a statement by a party opponent, b/c she is the one that wants it in. FRE 803(8) & (10) Public Records and Reports The ACN proposed a non-exclusive list of four factors it thought would be helpful in passing on this question: (1) the timeliness of the investigation; (2) the investigators skill or experience; (3) whether a hearing was held; and (4) possible bias when reports are prepared with a view to possible litigation Is a report than gives conclusions and opinions still admissible under 803(8)? Yes, from the holding in Beech Aircraft Corp. v. Rainey, where the court held that portions of investigatory reports otherwise admissible under FRE 803(8)(c) are not inadmissible merely b/c they state a conclusion or opinion. As long as the conclusion is based on a factual investigation and satisfies the FREs trustworthy requirement it should be admissible along with other portions of the report. Police Reports and Business Records FRE 803(8)(B) and 803(8)(C) do not extend to the reports of law enforcement personnel or evaluative reports if offered against the in a criminal case. United States v. Oats - Law enforcement reports are absolutely inadmissible against defendants in a criminal case. FRE 803(8)(c) does not compel the exclusion of documents properly admitted under FRE 803(6) where the authorizing officer or investigator testifies. i.e. if chemist does not show up, lab report cannot be admitted. United States v. Hayes - Rejected Oates and stated that the FRE 803(8)(B) provision was intended

to apply to observations made by law enforcement officials at the scene of the crime, and not to reports of routine matters made in non-adversarial settings United States v. Weiland that it has to be routine and non-adversial and the preparer has to be there to testify with all that the court may allow the information in FRE 807: Residual Exception A statement not specifically covered by FRE 803 or 804, but having equivalent circumstantial guarantees of trustworthiness, is not excluded by the hearsay rule if the court determines that (A) The statement is offered as evidence of material fact (B) The statement is more probative on the point for which it is offered than any other evidence which the proponent can procure through reasonable efforts and (C) The general purposes of these rules and the interest of justice will best be served by admission of the statement into evidence However, a statement may not be admitted under this exception unless the proponent of it makes known to the adverse party sufficiently in advance of the trial or hearing to provide the adverse party with a fair opportunity to prepare to meet it, the proponents intention to offer the statement and the particulars of it, including the name and address of the declarant To use this exception, the evidence has to be very unique; NOT covered by any of the FREs FL does not have a residual exception. (10/27/08 & 10/29/08) - Chapter 8: The Confrontation Clause and Hearsay The Confrontation Clause and Hearsay The 6th amendment provides that in all criminal prosecutions, the accused shall enjoy the right..to be confronted with the witnesses against him.. Question to ask o 1st look to see if it falls under the hearsay exception or exclusion o 2nd if not then look at the Confrontation Clause Constitution is high law but the FRE are a stricter law; evidence permitted by the FRE but forbidden by the confrontation clause must stay OUT and vice versa. Neither the Hearsay exceptions nor the Confrontation Clause will override each other. Route of Admissibility FRE 401 FRE 402 Evidence of an out of court Words or conduct

If evidence is hearsay as defined in 801(c)

If evidence is not hearsay under 801(c)

If evidence fits w/in no if evidence fits w/in exception/exclusion Exception from the hearsay rule from the hearsay rule Hence=inadmissible Hence=admissible under hearsay rules Hearsay If offered against in criminal case If offered in civil case or against prosecution

Raises Confrontation Clause Raises NO Confrontation Clause If declarant appears for cross-examination at trial (Crawford) If declarant is unavailable to testify, and had past chance to cross-examine (Crawford) If statement is not testimonial (Whorton) If forfeited confrontation rights by wrongdoing aimed at keeping declarant from testifying (Giles) (perhaps) f statement = dying declaration (Crawford) Else= Confrontation Clause violation Inadmissible Crawford distinguishes b/w testimonial and non-testimonial statements Testimonial is prior testimony at a preliminary hearing before a grand jury or at a former trial o Preliminary hearing, before a grand jury, or o At a former trial, and to o Police interrogations. o Ex parte in-court testimony or its functional equivalent (affidavits, custodial examinations o Prior testimony that the was unable to cross-x, or o Similar pretrial statements that declarants would reasonably expect to be used prosecutor FRE 403 Possibly Admissible

Primary purpose test what is the primary purpose of the questioning by the interrogator Non-Testimonial are causal remarks to an acquaintance, of hand remarks and statements in furtherance of a conspiracy (falling outside of confrontation clause protection) o Emergency calls (i.e 911) o Business records o Dying declarations (maybe)

The 3 formulations under Crawford Whether declarants would reasonable expect their statements to be used prosecutorially Testimonial-materials standard (which is now defunct) Whether statement were made under circumstances which would leans an objective witness reasonable to believe that the statement would be available for use at a later trial Forfeiture by Wrongdoing under Giles Means Procurement Problem 8.1: Deliver after Death: Woman believes that her husband is poisoning her and wrote a letter and gave it to her neighbor. Only a few weeks later did she die of poisoning and during her autopsy found the antifreeze in her system. Police arrested husband and charged him for her murder. The court should look at the letter as non-testimonial evidence because it was made out in the event that she dies. However it could testimonial because she wants the police to know everything. Problem 8.2 Confession in Blood: Rivera was charged with murder of Audrey. Prosecution wanted to admit evidence of a conversation between the and Caraballo, a nephew of Glanville, who was an acquaintance of the . The trial court admitted the statement. Did the admission of the testimony violate the s confrontation right? We want to get in Glanvilles statement in against the . It is also against Glanvilles penal interest b/c he was an accomplice. Glanville is not available for crossexamination. Defense will argue that it is an accomplice statement and therefore should be excluded, b/c generally accomplice statements are thought to be unreliable. The Prosecution will argue that it is reliable, b/c he said it to his nephew, and therefore it is non-testimonial. Exam do hearsay analysis first (800s). After hearsay analysissee if it a violation of the confrontation clauseunder CrawfordNOT Roberts. Look to see if it is testimonial or nontestimonial. Look at chart on page 453 (dotted lines we are not sure what analysis the court will use). Problem 8.3 Dove Interview: The victim was raped by her boyfriends boss and she reported this information the police. They then escorted her to the DOVE center where she signed a release and gave samples. During the examination the officer did not say a word but she had a conversation with the nurse. However sometime later the victim died from unrelated reasons but the believes that the conversation should not be let in b/c it would be against his 6th amendment right. In this case although the evidence is admissible hearsay based on 804(3) b/c the victim is dead and it would based on 804(4) Problem 8.4 Child was molested and repeated the story 4 different people and when trying to testify

she became scared. Would it violate the confrontation clause if the 4 people testified and the child did not against the Problem 8.4 Victim wants to recant b/c of domestic situation, at trial she says I dont know and such so the can be released. Now if the other occasions were in the Grand jury and other statements can the prosecution use that instead of her testimony at the actual trial. The Bruton Doctrine Confessions by Accomplices: the confrontation clause prohibited using against a the confession of an accomplice that he and the had committed the crime. Generally co-s confessions are not admissible. How does this relate to Coconspirators: This is a firmly rooted exception to hearsay and will generally be admissible. This is not a problem under the Bruton doctrine b/c coconspirators statements usually do not qualify as this when given to a police officer and generally do not have to be dealt with under Bruton. Bruton presents this question: May the trial court permit the jury to hear the out-of-court confession so long as the court carefully instructs the jury to consider the statement only against its maker? Problem 8.6: Dog O War (From dog Mauling Problems) Devan Hawkes, a special agent for the corrections department was called to testify that the Knollers were members of a white supremacy group and trained their dogs to fight. (What arguments to get the evidence in, what is the result under Bruton?) The evidence may not be reliable (comes from rumors of prison gangs).Rule: The dog of war statement is hearsay offered to prove the truth of the matter asserted. The best argument to get it in is that it is a co-conspirator statement b/c the letter being communicated was done in furtherance of the conspiracy b/c it encouraged trust and cohesiveness between the two. (FRE 803) Chapter 9: Lay Opinions and Expert Testimony FRE 701: Opinion Testimony by Lay Witnesses If the witness is not testifying as an expert, the witness testimony in the form of opinions or inferences is limited to those opinions or inferences which are (a) Rationally based on the perception of the witness and (b) Helpful to a clear understanding of the witnesss testimony or the determination of the fact in issue, and (c) Not based on scientific, technical or other specialized knowledge within the scope of FRE 702. FRE 701(a) demands, first of all, that a lay witnesses opinions be rationally based on her perception hence simply reaffirming the first-hand knowledge requirement FRE 602. FRE 701(b) requires in addition that lay opinions be helpful to the jurys factfinding. Most lay opinions convey reliable and material information (beyond what

jurors already know) clear this second hurdle. FRE 701(c) commands that a lay witness not invade the experts realm. FRE 701(c) if the testimony is specialized etc, must pass the muster under FRE 702 AND FRCP 26 and F.R.Crim.P. 16 include special disclosure requirements for expert testimony Problem 9.1 Prosecutions witness, an insurance claims investigator testifies about a conversation he had with the . When asked about a conversation with the , the investigator told the court that the sounded depressed. If defense counsel objects to the last answer, how should the court rule? Yes, this was the rationally based perception of the witness that does not require special knowledge. The judge should (and did) allow this as lay person testimony. Problem 9.2 Y is charged with selling cigarettes to a minor. The statute allows the to raise the affirmative defense that he reasonably believed the minor to be of age. Y seeks to admit testimony of other witnesses who will testify that she appeared to them to be 20 or 21 years old. Is this admissible? Yes, this opinion meets the qualifications of 701 and was admissible. Problem 9.3 s step-daughter was called on to testify that the powder found inside s shoes was cocaine. Daughter derived her opinion from her past experience with cocaine, and that she had looked at it and tasted it. Should the trial court have admitted this testimony over the s objection? This evidence was allowed in under 701 b/c of the witnesss particularized knowledge, however it would not be admitted in FL. Problem 9.4 The state wanted to use a witnesss testimony as an expert because he could depicter what the meaning behind the s work. The defense argued that he can not be a expert because he does not cryptography, so the court denied the states motion to tender him as a expert. However the court let him testify as a lay witness, would the evidence be admissible. The evidence would be admissible because based on the narrowly defined reading of the advisory committee notes, the court stated as long as the person knows of the specialty then they can testify to it. In this case, he learned the information and was able to translate the s code words. Expert Testimony FRE 702: Testimony by Experts If scientific, technical or other specialized knowledge will assist the trier of fact to understand the evidence or to determine a fact in issue, a witness qualified as an expert by knowledge, skill, experience, training or education, may testify thereto in the form of an opinion or otherwise, if (1) The testimony is based upon sufficient facts or data (2) The testimony is the product of reliable principles and methods, and (3) The witness has applied the principles and methods reliably to the facts of the case Analysis: (1) Is it helpful or relevant? (2) Is he/she qualified? How does the judge determine whether the expert is qualified will be based on FRE

104(a) voir dire of the expert to ascertain creditability at or before trial Motions in Limine What is beneficial about qualifying a witness as an expert? More faith from the jury Testify as to hearsay testimony To qualify as an expert the witness must have: Proper qualifications (by knowledge, skill, experience, training, or education) o FRE 702 Proper topic (must concern a topic that is beyond the ken of jurors) o FREs 702 and 704 Sufficient Basis (adequate factual basis for her opinions) o FREs 702 and 703 Relevant and Reliable Methods (the methods being applied to facts of the case) o FRE 702 FRE 403 challenge o FRE 403 weighing test Problem 9.5 A 34 yr-old CA. man confessed to smuggling 8 tons of marijuana into the US at estimated costs $4.8 million. Defense attorneys claimed that the only way the man knew the difference in the source of the drug was that someone had told him. However, he testified that he had smoked marijuana over 1,000 times and could identify by its appearance, smell, and high, whether it was Columbian or US. Is the language of FRE 701 broad enough to permit this witness to testify as a lay witness that the marijuana he smoked was imported from Columbia? Before you can get in a s statement, you have to have corroborating evidence; therefore, since there was no corroborating evidence, they could not charge this guy. Admissible under 701 and admissible under 702. Problem 9.6 An NYC police officer testified as an expert witness, in the field of narcotics trafficking, including . . . analysis of codes, words, and references used by narcotics traffickers. The trial court concluded that the officer was qualified for these 3 reasons. Extensive undercover narcotics work experience. He was an FBI and DEA wiretap techniques instructor and had listened to over 350 wiretaps. He was then allowed to give opinion testimony as to the meaning of intercepted phrases. Should the court have allowed the police officer to testify as an expert and give opinions? Yes, b/c all 3 factors of 702 are met. Testimony is based upon sufficient facts or data. The testimony is the product of reliable principles and methods. The witness has applied the principles and method reliably to the facts of the case. The testimony is based upon the facts of the instant case. (The intercepted conversations.) The officers experience and street knowledge from undercover work has shown him the methods of drug dealers. It is this knowledge that is the basis for his reliable methods of discerning covert speech. The witness applied his knowledge and experience from his teaching and undercover work to the facts of the case. No, FRE 702 requires 3 factors to be satisfied to admit expert opinion testimony. Overall, this probably would pass, but the opposing counsel would be given an opportunity to question the expert in voir dire. FRE 704: (Im)Proper Topics of Expert Testimony

Matters of Common Knowledge (FRE 702) Opinions on Law and opinions on ultimate issues (FRE 704) Opinions on creditability Opinions on eyewitness ID

Problem 9.7: Lewis Libby was being charged by a special prosecutor for lying to FBI investigators about conversation he had with some news reporters. Libby wanted to introduce testimony of an expert about how his memory could have faded. Should the trial judge have rule on the special prosecutors motion in limine to bar the expert testimony? No, the evidence should be let in based on FRE 702. The expert has the knowledge and requisite background to speak about the memory problems, the expert is using those reliable principles and has applied them to the case. Problem 9.8: Confusion? A normal person can distinguish between Macho and Match. We do not need an expert for this and even if it is a lay person, a jury knows the difference between these two marks. There is nothing in here that qualifies the witness as an expert. It should not come in as either lay opinion or expert testimony, b/c it is a big waste of time. Problem 9.9: Housing Ads: All housing ads had only white people in it. The suit was that the ads discriminated against black people. Expert was prepared to testify that all white advertising affects black markets. He was a doctor. Is the evidence admissible under FRE 702? Probably not. A lay person knows when it will make a person feel bad. If the expert could testify as to statistics through reliable scientific research, it could come in b/c they are showing how much the harm is. Opinions on Law and Opinions on Ultimate Issues FRE 704: Opinion on Ultimate Issue (a) Except as provided in (b), testimony in the form of an opinion or inference otherwise admissible is not objectionable because it embraces an ultimate issue to be decided by trier if fact (b) NO expert witness testifying with respect to the mental state or condition of a in a criminal case may state an opinion or inference as to whether the did or did not have the mental state of condition constituting an element to the crime charged or of a defense thereto. Such ultimate issues are matters for the trier of fact alone Expert opinion must be helpful to the trier of fact and cannot tell the jury what result to reach So, although FRE 704 specifically permits experts o opine on an ultimate issue to be decided by the trier of act (except certain opinions on s mental state in a criminal case), there is a line past which experts may not go Problem 9.10: A 60yr old retired minister is challenged a conviction of possession a drug with intent to manufacture the drug. The court show some purchases made by the elderly couple. At trial chemist outs all those ingredients together stating that they were going to make this drug. Admissible testimony? Yes it could be admissible b/c it satisfies FRE 702 b/c the trier of fact

would not be able know such information without expert testimony. Opinions on Credibility Problem 9.11 Thomas Nimely was shot and paralyzed by an NYC and he later brought a federal civil suit against the officer, dept and the city. The central issue was when and how the was shot. The expert testimony of the defense stated that the officers may have lied etc. should the trial court have overruled the s objection to this testimony? No, b/c the trier of fact is believing the expert testimony as if they knew everything. The Golden Rule: You cannot ask the jury to put themselves in the shoes of the victim or the or something else at ANYTIME during the trial Opinions on Eyewitness Identification FRE 703 and 705: Proper Bases of Opinion Testimony FRE 703: Bases of Opinion Testimony by Experts The facts or data in the particular case upon which an expert bases an opinion or inference may be those perceived by it made known to the expert at or before the hearing. If of a type reasonably relied upon the experts in the particular field in forming opinions or inferences upon the subject, the facts or data need not be admissible in evidence in order for the opinion or inference to be admitted. Facts or data that are otherwise inadmissible shall not be disclosed to the jury by the proponent of the opinion or inference unless the court determines that their probative values in assisting the jury to evaluate the experts opinion substantially outweighs their prejudicial effect FRE 403 is applied only if the court allows for the proponent to enter in the facts or data to show to the jury FRE 705: Disclosure of Facts or Data Underlying Expert Opinion The expert may testify in terms of opinion or inference and give reason therefor without first testifying to the underlying facts or data, unless the court requires otherwise. The expert may in any event be required to disclose the underlying facts or data on cross examination.

Two things distinguish expert opinions from lay opinions the background knowledge that informs them and the specific acts on which they rest. Experts Can Rely On: Facts they personally observed Testimony given at trial (usually given in the form of hypotheticals) o Ex: Assume Witness G was accident when they said Facts made known to the expert before trial (HEARSAY) o The hearsay experts rely on is admitted under 803(18), a learned treatise or document in the field that is generally excepted

Problem 9.12 Police officer is asked to testify as an expert to the fact that criminals often conceal weapons in the engine compartment instead of the glove compartment, so that if they were found, it was easier to claim they did not know they were there. Officer based his experience of this practice on past experience and questioning of 50-60 inmates. While this is hearsay, it could be admitted if it was reasonably relied upon by experts in the field in forming opinions or inferences upon the subject under FRE 703. Most likely this is not admissible since talking to inmates about their past criminal practices is not a reliable method in the field. Most likely, if let in, it would not be an abuse of discretion. **It is the judges role to decide whether or not the evidence comes in. Problem 9.13 Plaintiff alleging medical negligence brings in an expert who testifies that in reviewing a three page consultants report, that a cannula being used during the surgery had negligently come out. The doctor reporting this information stated that he had never seen a statement like this written in a chart and did not know the doctor who had written. This was not admissible at trial since there was no proof that this statement was regularly relied upon. Judge would have to make this determination under 104(a). Might have come in under 801(d)(2)(b), but this was never argued. In Re Melton A Doctor was asked if released would he be likely to injure himself or others? In reaching his conclusion, the doctor relied upon testimony from relatives who believed he would be a danger. On appeal, Melton argued that this was inadmissible hearsay. The court held that this was not even an issue, since doctors often ask those who know the patient best their opinion in deriving their conclusion. The court believed the doctor correctly used the mothers testimony and the court correctly allowed it in, though the judge gave a limiting instruction. Assessing the Reliability of Expert Scientific Testimony Frye Test: Must have sufficiently established to have gained general acceptance in the particular field in which it belongs Daubert Test (5 factors): (this inquiry is a flexible one) o Whether it has/or can be tested; o Whether it has been reviewed by colleagues or published; o Known or potential rate of error of theory or technique; and o Means or standards of controlling the techniques operation; and o General acceptance in the relevant scientific community.

The judge would decide if these factors would work under 104(a) to allow the testimony into evidence Court is now the gatekeeper so they have to be mindful of FRE 703, 706 and 403 Two-Part Daubert Test: o Must determine nothing less than whether the experts testimony reflects scientific knowledge, whether their work product amounts to good science. o The proposed expert testimony is relevant to the task at hand, i.e., that it logically advances a material aspect of the proposing partys case.

Problem 9.15 Mother sought to admit expert testimony as to the value of her sons life in an action for wrongful death. The first point of attack for opposing counsel is 1) is the person an expert 2) is the testimony relevant and 3) will it assist the jury. In this case, the judge did not allow the testimony finding that it did not meet Daubert factors and was therefore not reliable to the jury. If this was hearsay it has to be information that was reasonably relied upon in the field. Problem 9.16 Chapter 10: Authentication, Identification and The Best Evidence Rule FRE 901: Requirement of Authentication or Identification (a) General Provision. The requirement of authentication or identification as a condition precedent to admissibility is satisfied by evidence sufficient to support a finding that the matter in question is what its proponent claims. (b) Illustrations. By way of illustration only, and not by way of limitation, the following are examples of authentication or identification conforming with the requirements of this rule: (1) Testimony of witness knowledge. Testimony that a matter is what it is claimed to be (2) Non-expert opinion on handwriting. Non-expert opinion as to the genuineness of handwriting, based on familiarity not acquired for purposes of the litigation. (3) Comparison by tried or expert witness. Comparison by the trier or fact or by expert witnesses with specimens which have been authenticated (4) Distinctive characteristics and the like. Appearances, contents, substance, internal patterns or other distinctive characteristics, taken in conjunction with circumstances (5) Voice Identification. Identification of voice, whether heard firsthand or through mechanical or electronic transmission or recording, by opinion based upon hearing the voice at any time under circumstances connecting it with the alleged speaker (6) Telephone conversation. Telephone conversation, by evidence that a call was made to the number assigned at the time by the telephone company to a particular person or business, if (A) in the case of a person, circumstances, including self-identification, show the person answering to be the one called, or (B) in the case of a business, the call was made to a place of business and the conversation related to business reasonably transacted over the telephone (7) Public records or reports. Evidence that a writing authorized by law to be recorded or filed and in fact recorded or filed in a public office, or a purported public record, report, statement or data compilation, in any form, is from the public office where items of this nature are kept (8) Ancient documents or data compilation. Evidence that a document or data compilation, in any form, (A) is in such condition as to create no suspicion concerning its authenticity, (B) was in a place where it, if authentic would likely be, and (C) has been in existence 20 years or more at the time it was offered (9) Process or system. Evidence describing a process or system used to

produce a result and showing that the process or system produces an accurate result. (10) Methods provided by statute or rule. Any method of authentication or identification provided by Act of Congress or by other rules prescribed by the Supreme Court pursuant to statutory authority. Use 104(b) to determine authentic before using an exception. With FRE 901, 902 still have to look at hearsay and the best evidence rule FRE 902: Self-authentication Extrinsic evidence of authenticity as a condition precedent to admissibility is not required with respect to the following: (1)Domestic public documents under seal (2)Domestic public documents not under seal (3) Foreign public documents (4) Certified copies of public records (5) Official publication (6) Newspapers and periodicals (7) Trade inscriptions and the like (8) Acknowledge Documents (9) Commercial paper and related documents (10) Presumptions under Acts of Congress (11) Certified domestic records of regularly conducted activity (12) Certified foreign records of regularly conducted activity Although all of these documents are seen as hard to duplicate, the ACN states that in no instance is the opposite party foreclosed from disputing authenticity Chain of Custody: It is not necessary to establish every link in the chain of custody in non-fungible evidence. A missing link in the chain of custody goes to weight not admissibility. In the case of fungible evidence such as cocaine, there are stricter guidelines to meet in assuring that it is the same item, in substantially the same condition. Proof of a perfect chain of custody consists of testimony be each person who had custody of the item from the moment it was seized from the until its delivery to the courtroom. Documents Problem 10.1 In United States v. Vigneau, the 1st circuit FREd that evidence of Western Union to send money forms used to transmit funds cross-country was wrongfully admitted. Whoever had filled out the forms had identified the sender as Vigneu. The court held that this statement was hearsay, but added that if there were evidence to corroborate that the person writing on the form was Patric Vigneau, it would fit in as a statement by a party opponent under FRE 801(d)(2). The Western Union forms from Vigneau survived. If you were the prosecutor, how could you establish that Vigneau was the party that wrote the forms? Handwriting analysishave an expert witness compare handwriting under 901(b)(3). It is not necessary that it be an expert, the jury can decide for themselves, whether the handwriting is the same. Is giving a handwriting sample a violation of the

5th amendment? 5th amendment only applies to testamentary. Under 903(b)(2), someone who is familiar with the defendants handwriting can testify. The evidence is classified as Documentary evidence, and could be authenticated by laying a foundation of preliminary evidence sufficient to sustain a finding that the matter is what it purports to be. (It could be authenticated by direct identification testimony personal knowledge, distinctive markings or characteristics, or chain of custody) Problem 10.2 FRE 901(b)(8) authentication of an ancient document may be supplied by demonstrating that a document is in such condition as to create no suspicion concerning its authenticity, was in a place where, if authentic, it likely would be, and has been in existence 20 years or more at the time it is offered. Ancient documents are admissible into evidence as an exception to the hearsay rule. Problem 10.3: Anonymous Note II: Consider the Facts in U.S. v. Mitchell: between 9:00 and 9:15 am, on 9/12/91, two men waited in a check cashing store. An armored truck made a delivery of currency to the store, the assailants attached the delivery man and robbed him of an excess of $20,000. The two men fled the scene is a beige car driven by a third person. At 10:00 am, the police received an anonymous note from the front seat that stated light green. They changed cars and this is the other car. It was discovered that the car belonged to Mitchells then fianc. A handwriting expert, a fingerprint expert, and a chemist examined the note and said that they had no way of identifying who wrote it. Question: on the s objection that the evidence was inadmissible b/c it had not been properly authenticated, how should the court have FREd? Answer: The court should not let it in, b/c it does not satisfy the general provision of 901(a) in order to authenticate it, you do not have to prove who wrote it. (There may be an argument to let it in). Phone Calls Problem 10.4: Star 69: On the morning of 10/30/93, a bearded man arrived at the Pizza Hut wearing the Pizza Hut uniform asked for the manager by name and persuaded employees to let him in. When the manager came in he told her he was training and handed her an application folder. Inside the folder was a message that read, just give me the money and you wont get shot. The manager complied and as he was leaving he showed her the handle of his gun. Manager then called the police and her regional manager. Shortly after the incident, the man called the store to make sure nobody had been fired over the incident. The stores phone has #69 on it that allowed them to redial the last number to call and it was the South Shepard Pizza Hut store. The police went to the store, found a man fitting the mans description, and recovered a .38 hand-gun and $284. Assuming that the man was not identifiable and that there was no evidence that he normally wore a beard. If the prosecutor tries to admit the words the man said on the phone call are they admissible? Under 901(b)(9) the call would likely be admissible, b/c the man did not personally identify himself, and it was not in relation to the business. (The state must show that he works at the pizza hut, 901(b)(4)(2) value of other circumstances surrounding the facts; the circumstances found in the car would likely be able to authenticate the video) Photographs Problem 10.5: Staged Photo: In State v. Kelley, the s were convicted of having shot C.L. Jenkins through a window of his house with the intent to kill him. Jenkins, though injured survived. A

staged photo of him was taken the next morning showing Mr. Jenkins in the position he was in when he was shot. Prosecution wants to admit the pictures. Assume that Helm was in the room with Jenkins at the time he was shot. If the prosecutor wants to admit it to show Mr. Jenkinss position and the shooters view, what is the defenses best argument? The best evidence is that is does not work as demonstrative evidence, b/c the photo did not accurately demonstrate material facts. (a reconstruction may not be an accurate depiction of the events.)(if it was going to be admitted to show Jenkins position at the time that he was shot, then it might be admissible.) Assume: Helm was not present at the time of the attack, but that all the other facts remain the same. s Best Argument: there is not sufficient evidence to prove that the matter is question is what the proponent claims since Helm was not present at the time of the shooting see 901(a) Problem 10.6: Have the experts testify about the CGA and then the prosecution can offer it as a pictorial illustration of the experts testimony Silent Witness Theory That photographic evidence may be admitted upon proof of the reliability of the process, which produced the photograph or videotape. The relevant photographic evidence may be admitted into evidence on the silent witness theory after the judge has considered: 1. Evidence of time and date of the photographic evidence; 2. Evidence of editing or tampering 3. Operating condition and capability of the equipment producing the photographic evidence as it relates to the accuracy and reliability of the photographic product; 4. Procedure employed as it relates to the preparation, testing, operation, and security of the equipment used to produce the photographic product, including the security of the product itself; and 5. Testimony identifying the relevant participants depicted in the photographic evidence The Best Evidence Rule FRE 1001: Definitions For purposes of this article the following definitions are applicable (1) Writing and recordings. writings and recordings consist of letters, words, or numbers or their equivalent, set down by handwriting, typewriting, printing, photostating, photographing, magnetic impulse, mechanical pr electronic recording or other form of data compilation (2) Photographs. photographs include still photographs, x-ray films, video tapes and motion pictures (3) Original. An original of a writing or recording is the writing or recording itself or any counterpart intended to have the same effect by a person executing or issuing it. An original of photograph includes the negative or any print thereform. If data are stored in a computer or similar device any printout or other output readable by sight, shown to reflect the data accurately is an original (4) Duplicate. A duplicate is a counterpart produced by the same impression as the original, or from the same matrix, or by means of photography, including enlargements and miniatures or by mechanical or electronic re-recording, or by chemical reproduction, or by their equivalent technique which accurately reproduces the original

FRE 1001(3) original includes a photograph made from the same negative is an original, as is a new print made from an old plate or a new printout of stored computer data. Carbon copy is an original and the carbonless copy of a credit card receipt qualifies as well FRE 1001(4) a photograph or photocopy of a document is a duplicate, as is a rerecording of an old tape or a photographic print made from another print. Copies produced manually, whether handwritten or typed, are not within the definition. FRE 1002: Requirement of Original To prove the content of a writing, recording, or photograph, the original writing, recording, or photograph is required, except as otherwise provided in these rules of by Act of Congress. The policy behind the rule is that it is aimed to assure that whenever possible, the jury has access to an original or duplicate if a case turns on the content of a writing, recording or photograph. Best Evidence Rule: In proving the terms of a writing, where the terms are material, the original writing must be produced unless it is shown to be unavailable for some reason other than the serious fault of the proponent. (1) Document has independent legal significance (i.e. wills, contracts); (2) FRE 1003: Admissibility of Duplicates A duplicate is admissible to the same extent as an original unless (1) a genuine question is raised as to the authenticity of the original or (2) in the circumstances it would be unfair to admit the duplicate in lieu of the original Admissibility of other Evidence of Contents The original is not required, and other evidence of the contents of a writing, recording or photograph is admissible if (1) Original lost or destroyed. All originals are lost or have been destroyed, unless the proponent lost or destroyed them in bad faith; or (2) Original not obtainable. No original can be obtained by any available judicial process or procedure or (3) Original in possession of opponent. At a time when an original was under the control of the party against whom offered, that party was put on notice, by the pleadings or otherwise, that the contents would be subject of proof at the hearing, and that the party does not produce the original at the hearing or (4) Collateral Matters. The writing, recording or photograph is not closely related to a controlling issue The ACN state that the basic of this rule is requiring of the production of the original as proof of contents but if failure to produce the original is satisfactory explained then secondary evidence is admissible

FRE 1004:

FRE 1005:

Public Records The contents of an official record, or of a document authorized to be recorded or filed and actually recorded or filed, including data compilations in any form, if otherwise admissible, may be proved by copy, certified as correct in accordance with FRE 902 or testified to be correct by a witness who has compared it with the original. If a copy which complies with the foregoing cannot be obtained by the exercise of reasonable diligence, then other evidence of the contents may be given Summaries The contents of voluminous writings, recordings, or photographs which cannot conveniently be examined in court may be presented in the form of a chart, summary, or calculation. The originals, or duplicates, shall be made available for examination or copying, or both, by other parties at reasonable time and place. They may order that they be produced in court This rule only shows the practicable means of making the contents available to the judge and jury

FRE 1006:

FRE 1007:

Testimony or Written Admission of Party Contents of writings, recordings or photographs may be proved by the testimony or deposition of the party against whom offered or by that partys written admission, without accounting for the nonproduction of the original. Functions of Court and Jury When the admissibility of other evidence of contents of writings, recordings or photographs under these rules depends upon the fulfillment of a condition of fact, the question whether the condition has been fulfilled is ordinarily for the court to determine in accordance with FRE 104. However when an issue is raised (a) whether the asserted writing ever existed, or (b) whether another writing, recording or photograph produced at trial is the original or (c) whether other evidence of contents correctly reflects the contents, the issue is for the trier of fact to determine as in the case of other issues of fact

FRE 1008:

Problem 10.7 Instead of admitted the transcript (the most likely method of proving testimony), the prosecutor questioned a witness to testify as to Ls earlier testimony which was perjury. The witness refused to testify saying that the prosecutor should bring in the transcripts. Here the contents of the testimony are at issue and therefore prosecutor more than likely should have brought in the official transcripts to prove the content of the testimony, however it does implicit the BER. Problem 10.8 A card or a witness testifying to a license plate number? Prosecutors witness can remember the number, but defense attorney wants prosecutor to admit the card, the license plate number was written on. Here the contents of the card are not at issue and therefore it is not necessary to bring in the BER. The prosecutor is simply trying to show what the license plate number was. Problem 10.9 GPS found on the boat, prosecution asked the officer to testify to its back tracking in court but never brought in the GPS from the boat. (a) The trial should have ruled against the

prosecution and stated that they have to follow the best evidence rule because the GPS is a recording that has to be shown to the jury. That the actual GPS and all the paperwork to show the backtracking should be shown in court. (b) Now if a co-conspirator destroyed the evidence then the best evidence rule would not apply, that copies or the testimony of the officer would suffice because of the destruction of the original evidence. Chapter 11: Privileges & FL distinctions FRE 501: General Rule Except as otherwise required by the Constitution of the United States of provided by Act of Congress or in rules prescribed by the Supreme Court pursuant to statutory authority, the privilege of a witness, person, government, State, or political subdivision thereof shall be governed by the principles of the common law as they may be interpreted by the courts of the United States in the light of reason and experience. However, in civil actions and proceedings, with respect to an element of a claim or defense as to which State law supplies the rule of decision, the privilege of a witness, person, government, State, or political subdivision thereof shall be determined in accordance of State Law. Federal court + diversity = state common law Federal court + fed question = federal common law Atty/Client OR Physiotherapist/Patient OR Husband/Wife Privilege: A privilege is a rule of law that, to protect a particular relationship or interest, either permits a witness to refrain from giving testimony or otherwise could be compelled to give, or permits someone (usually one of the parties) to prevent the witness from revealing certain information. Problem 11.1:

También podría gustarte